Sunteți pe pagina 1din 31

Test 1.

ª vuelta Distancia
Pediatría

Test 1.ª vuelta Distancia

Pediatría
1. Señale la afirmación correcta respecto al test de Apgar: 2) La clavícula es el hueso que se fractura con mayor frecuencia
durante el parto.
1) Se debe practicar únicamente a los RN patológicos o supues- 3) El llamado “nódulo de Stroemayer” suele requerir trata-
tamente patológicos. miento quirúrgico.
2) Entre los parámetros que se valoran se encuentra la frecuencia 4) La rotura hepática no siempre precisa tratamiento quirúrgico.
respiratoria. 5) Las fracturas craneales más frecuentes en el parto son las
3) Se debe realizar a los 1-5-10 minutos de vida. lineales.
4) Un test de Apgar de 4, al minuto de vida, implica un alto
riesgo de parálisis cerebral. 5. ¿Cuál de los siguientes fenómenos dermatológicos que podemos
5) La palidez cutánea generalizada supone una puntuación encontrar en un neonato NO debe sugerirnos una patología
de 1 en el test de Apgar. subyacente?

2. RN que, al minuto de vida, presenta FC de 110 latidos por 1) Manchas hipopigmentadas geográficas.
minuto, acrocianosis con esfuerzo respiratorio ausente, así como 2) Ictericia precoz en las primeras 24 horas.
hipotonía y leve mueca al introducir la sonda de aspiración. La 3) Nevus flammeus.
puntuación de Apgar será: 4) Melanosis pustulosa.
5) Manchas café con leche.
1) 1
2) 2 6. Ante un neonato a término que tras cesárea presenta precoz-
3) 3 mente taquipnea y cianosis que desaparece con pequeñas
4) 4 cantidades de oxígeno, en el que la auscultación pulmonar es
5) 5 normal y en el que la radiografía muestra marcas vasculares
prominentes y líquido en las cisuras sin broncograma aéreo,
3. ¿Cuál de las siguientes afirmaciones sobre los cuidados del sospecharemos:
recién nacido prematuro es FALSA?
1) Enfermedad de membrana hialina (EMH).
1) La alimentación debe iniciarse siempre por sonda nasogástrica 2) Taquipnea transitoria del recién nacido.
en los menores de 32 semanas de edad gestacional, por la 3) Síndrome de aspiración meconial (SAM).
falta de coordinación entre succión, deglución y respiración. 4) Persistencia de la circulación fetal (PCF).
2) En los casos que necesiten tratamiento con eritropoyetina, 5) Displasia broncopulmonar (DBP).
debe iniciarse a la vez tratamiento con hierro vía oral.
3) Las necesidades de líquidos son mayores que en los neonatos 7. RN de 35 semanas de edad gestacional que en las primeras 12
a término, tanto más cuanto más prematuro sea el recién horas de vida presenta taquipnea, quejido audible y cianosis
nacido. progresiva, refractaria a la administración de oxígeno. A la
4) Debe aumentarse el intervalo de dosis de todos los medi- auscultación encontramos crepitantes, sobre todo en bases
camentos de eliminación renal. pulmonares. En la gasometría aparece hipoxemia progresiva,
5) Hay que realizar a todos ecografía transfontanelar aunque aumento de la PCO2 y acidosis metabólica. ¿Qué radiografía de
no tengan alteraciones en la exploración neurológica. tórax esperaría encontrar en este momento?

4. Señale la afirmación FALSA respecto a las lesiones producidas 1) Normal.


por traumatismos obstétricos: 2) Infiltrado de patrón reticulogranular con broncograma aéreo.
3) Hiperinsuflación y aumento de las marcas vasculares, sin
1) El cefalohematoma es una hemorragia subperióstica, y está broncograma.
delimitado por las suturas craneales. 4) Patrón de esponja.

CTO Medicina • C/Francisco Silvela, 106 • 28002 - Madrid • Tfno. (0034) 91 782 43 30/33/34 • E-mail: secretaria@ctomedicina.com • www. ctomedicina.com 1
Test 1.ª vuelta Distancia
Pediatría
5) Borde de ambos pulmones colapsados. 5) A pesar de que pueda retirarse la administración de O2 antes
de salir de la unidad de cuidados intensivos, el pronóstico
8. En el tratamiento de la EMH se incluyen las siguientes medidas, para la mayoría de estos pacientes es malo, presentando
EXCEPTO: en la adolescencia obstrucción de las vías respiratorias e
hiperreactividad bronquial.
1) Administración de surfactante endotraqueal.
2) Antibioterapia. 12. Recién nacido a término que presenta dificultad respiratoria
3) Administración de oxígeno y ventilación mecánica, si es inmediata, con importante tiraje intercostal y supraesternal,
preciso. taquipnea, tórax hiperinsuflado y abdomen excavado. En la
4) Prostaglandinas. auscultación pulmonar destaca abolición casi total del mur-
5) Control del aporte de líquidos. mullo vesicular en el hemitórax izquierdo, con tonos cardíacos
desplazados a la derecha. Señale la respuesta verdadera acerca
9. Un niño de 42 semanas de gestación, arrugado, pálido, hipotó- de la patología que sospecha:
nico, apneico y cubierto de líquido amniótico verdoso. Respecto
al cuadro que puede padecer este paciente, señale la opción 1) En más del 50% de los casos hay otras malformaciones
FALSA: asociadas.
2) El tratamiento es quirúrgico, y debe realizarse lo antes posible,
1) La primera maniobra en su reanimación debe ser la aspiración tratando después las alteraciones que haya asociadas.
de tráquea bajo visión laringoscópica. 3) El diagnóstico diferencial con malformaciones pulmonares
2) Es un patología típica de RNPT y RNT. quísticas se hace con ecografía.
3) E. coli es el germen más frecuente de infección bacteriana 4) El reflujo gastroesofágico es una secuela rara tras la cirugía.
en estos pacientes. 5) Los afectados suelen desarrollar hipertensión pulmonar a
4) En la Rx de tórax es típico encontrar un patrón de atrapa- partir del nacimiento, que en los casos graves condiciona
miento aéreo. su pronóstico.
5) Esta patología produce una llamativa hipertensión pulmonar.
13. RNPT de 29 semanas de edad gestacional sufre distrés respi-
10. Recién nacido de 38 semanas que tras expulsivo dificultoso ratorio desde el nacimiento. A las 36 horas de vida presenta
presenta el brazo izquierdo pegado al tronco junto con cierta hipotensión, bradicardia, cianosis y la fontanela se encuentra
rotación interna de dicho miembro. Señale la respuesta verda- abombada. En el hemograma presenta 15.000 leucocitos, neu-
dera respecto a la lesión obstétrica que sospecha en este recién trófilos 45%, linfocitos 42%, cayados 0%, plaquetas 180.000,
nacido: hematocrito 30%. ¿Qué prueba complementaria consideraría
más adecuada en este momento?
1) El reflejo de Moro no estará presente en el lado izquierdo.
2) Se debe a una lesión en las raíces inferiores del plexo braquial. 1) Punción lumbar y análisis del LCR.
3) Este tipo de parálisis es menos frecuente que la parálisis de 2) Ecocardiograma.
Klumpke. 3) Ecografía cerebral.
4) Es característica la presencia de mano caída. 4) TC craneal.
5) Suele asociarse con síndrome de Horner. 5) Rx tórax.

11. Un RN de 32 semanas, con antecedente de enfermedad de 14. Un RN pretérmino de 1.800 g de peso, con antecedentes de
membrana hialina sometido a ventilación mecánica con altas hipoxia perinatal, a los 10 días de vida presenta distensión
concentraciones de oxígeno, tiene actualmente un mes de vida abdominal importante, vómitos y deposiciones hemorrágicas.
y sigue dependiendo del respirador para mantener adecuadas En la Rx de abdomen se observa edema de asas intestinales,
saturaciones de oxígeno. En los últimos días presenta además con un patrón en “miga de pan” y presencia de gas en la pared
oliguria con edemas en miembros inferiores y se palpa el intestinal. ¿Cuál sería la opción terapéutica más adecuada en
hígado a 5 cm bajo el reborde costal derecho. En la Rx de tórax este paciente?
se observa una imagen de pequeñas zonas redondas y claras
que alternan con otras de mayor densidad. En relación con 1) Enema de solución hiperosmolar.
la patología que presenta este paciente, señale la respuesta 2) Alimentación enteral a débito continuo a través de una
FALSA: sonda nasogástrica.
3) Actitud expectante y vigilancia cuidadosa, ante la posibilidad
1) Se consideran factores de riesgo para su desarrollo tener de perforación intestinal.
distrés respiratorio severo que precise largos períodos de 4) Se debe suspender la alimentación enteral y pautar fluido-
administración de O2. terapia y antibioterapia i.v.
2) Las principales causas de muerte de estos pacientes son la 5) La presencia de neumatosis intestinal aconseja la interven-
bronquiolitis necrotizante y fallo cardíaco derecho. ción quirúrgica urgente.
3) El tratamiento incluye broncodilatadores, diuréticos y dexa-
metasona. 15. RN de 38 semanas de edad gestacional presenta distensión
4) La nefrolitiasis puede ser una complicación de estos abdominal y vómitos biliosos durante el primer día de vida. En
pacientes, secundaria al uso de diuréticos y a la alimentación el examen físico se palpan cordones duros que siguen el marco
parenteral. cólico. Una radiografía en bipedestación muestra una masa en

CTO Medicina • C/Francisco Silvela, 106 • 28002 - Madrid • Tfno. (0034) 91 782 43 30/33/34 • E-mail: secretaria@ctomedicina.com • www. ctomedicina.com 2
Test 1.ª vuelta Distancia
Pediatría
“pompa de jabón” en cuadrante inferior derecho. El tratamiento 20. Respecto a la anemia en el período neonatal, señale cuál de las
inmediato más apropiado es: siguientes afirmaciones es FALSA:

1) Antibioterapia. 1) La anemia fisiológica es más intensa y más precoz en los


2) Supositorios de glicerina. RNT que en los RNPT.
3) Enema de solución hiperosmolar. 2) El cuadro de anemia fisiológica neonatal está en relación con la
4) Estimulación rectal. hemólisis de glóbulos rojos y con un déficit de eritropoyetina.
5) Cirugía. 3) En los RNPT está indicada la administración de suplementos
de hierro a partir de los 2 meses de vida.
16. Señale la afirmación FALSA, de entre las siguientes, respecto a 4) La anemia fisiológica puede precisar transfusión en algunos
la ictericia fisiológica: casos.
5) La betatalasemia no se manifestará antes de los 6 meses de
1) Las cifras máximas de bilirrubina suelen alcanzarse a los 3-4 vida.
días.
2) La ictericia en los RNPT suele ser de inicio algo más tardío y 21. Señale la afirmación FALSA, de entre las siguientes, respecto a
más prolongada que en los RNT. la isoinmunización Rh:
3) Puede comenzar en las primeras 24 horas de vida en el RNT.
4) Su duración suele ser inferior a 10-15 días. 1) La enfermedad hemolítica grave suele afectar al segundo hijo.
5) En los RNPT pueden alcanzarse cifras de hasta 14 mg/dl. 2) Suele cursar con ictericia, que aparece en las primeras 24
horas de vida.
17. En relación a la ictericia por lactancia materna o síndrome de 3) Debe hacerse profilaxis con gammaglobulina específica
Arias es FALSO que: anti-D antes de que pasen 72 horas del parto, cuando el
test de Coombs directo es positivo.
1) Su frecuencia es aproximadamente de 1 por cada 200 RN 4) Cuando las cifras de bilirrubina superan los 20 mg/dl, es
que reciben lactancia materna. necesario realizar una exanguinotransfusión.
2) Es una ictericia que aparece a la mitad o final de la primera 5) El 90% de los casos de incompatibilidad se deben al antí-
semana, alcanzando su máximo en la tercera. geno D.
3) Se pueden alcanzar cifras de hasta 30 mg/dl, de ahí el riesgo
de kernicterus en estos neonatos. 22. Respecto a la hipoglucemia del hijo de madre diabética, señale
4) En la patogenia se implican varias sustancias presentes en la cuál de las siguientes afirmaciones es FALSA:
leche materna como pregnanodiol, ácidos grasos de cadena
larga y glucuronidasas. 1) Se observa en el 75% de los hijos de madres diabéticas y en
5) Cuando se suspende la lactancia materna durante 2-4 el 25% de los hijos de madres con diabetes gestacional.
días se produce una rápida disminución de la bilirrubina 2) Aparece en las primeras 24 horas de vida y suele ser asinto-
sérica. mática.
3) Se piensa que es debida a hiperinsulinismo fetal.
18. En relación a la enfermedad hemolítica por isosensibilización 4) La probabilidad de que el RN presente hipoglucemia es más
AB0, señale la FALSA: elevada cuanto más altas se encuentren las cifras de glucosa
en sangre de cordón.
1) La madre suele ser 0. 5) Se trata mediante infusión i.v. de bolos de glucosa y debe
2) Los anticuerpos causantes suelen ser de la clase IgG. retrasarse la alimentación oral para minimizar el riesgo de
3) Pueden resultar afectos los productos del primer embarazo. hipoglucemia de rebote.
4) El antígeno más frecuentemente implicado es A1.
5) La enfermedad se agrava en los embarazos siguientes, si no 23. NO es propio del RN hijo de madre diabética:
hacemos profilaxis.
1) Megalencefalia.
19. Un lactante de una semana es llevado a Urgencias en estado de 2) Síndrome asfíctico.
coma con hemorragias en la retina y palidez importante. Entre 3) Hiperbilirrubinemia.
sus antecedentes destaca que el parto tuvo lugar en el domi- 4) Síndrome de regresión caudal.
cilio, no presentó complicaciones posteriores y está recibiendo 5) Cardiomiopatía.
lactancia materna. Señale lo FALSO respecto a este cuadro:
24. En la sepsis del RN, los gérmenes más frecuentes son:
1) Se debe a un déficit de los factores dependientes de
vitamina K. 1) Klebsiella.
2) Suele iniciarse a partir de las primeras 24 horas. 2) E. coli y estreptococos del grupo A.
3) Es más frecuente en hijos de madres que han tomado feni- 3) E. coli y estreptococos del grupo B.
toína o fenobarbital. 4) Estafilococo.
4) Las complicaciones hemorrágicas son menos frecuentes en 5) Gardnerella vaginalis.
niños que toman LM que en los que reciben biberón.
5) Se trata con la administración de vitamina K y/o plasma 25. ¿Qué haría usted ante un RN de madre con hepatitis B activa en
fresco congelado. el momento del nacimiento?

CTO Medicina • C/Francisco Silvela, 106 • 28002 - Madrid • Tfno. (0034) 91 782 43 30/33/34 • E-mail: secretaria@ctomedicina.com • www. ctomedicina.com 3
Test 1.ª vuelta Distancia
Pediatría
1) Observar la aparición de los síntomas y luego tratar. 30. Un RN hijo de madre prostituta presenta en el 7.º día de vida un
2) Extraer sangre de cordón para estudiar transaminasas. cuadro de fiebre con letargia y rechazo de las tomas así como
3) Exanguinotransfusión inmediata, si la madre tiene el antí- convulsiones. A la exploración destaca hepatoesplenomegalia
geno HBe positivo. con fontanela anterior abombada y lesiones vesiculares en ca-
4) Poner una dosis bimensual de gammaglobulina. lota craneal. ¿Cuál sería la causa más probable de este cuadro?
5) Iniciar vacunación y gammaglobulina en paritorio.
1) Infección por VIH.
26. Un RN de bajo peso para su edad gestacional presenta un 2) Infección por CMV.
exantema petequial, hepatoesplenomegalia y coriorretinitis. En 3) Infección por VHS tipo I.
la TC craneal se observan calcificaciones periventriculares sin 4) Infección por VHS tipo II.
otros hallazgos. Con respecto al cuadro que parece presentar 5) Exantema tóxico del RN.
este paciente, señale la verdadera:
31. Un recién nacido de 3 semanas de vida que desprendió el cordón
1) Es la segunda causa más frecuente de infección viral con- umbilical a los 16 días y cuyo ombligo no ha cicatrizado, pre-
génita. senta una pequeña tumoración sonrosada con secreción sero-
2) El 90% de los recién nacidos infectados estarán asintomá- mucosa con un pH de 7,4. ¿Cuál de los siguientes diagnósticos
ticos al nacer. le parece más probable?
3) La manifestación más tardía y característica es una hipoacusia
de transmisión. 1) Onfalocele.
4) La afectación ocular típica es una retinitis pigmentosa. 2) Granuloma umbilical.
5) Estos niños suelen desarrollar macrocefalia evolutivamente. 3) Quiste o persistencia del uraco.
4) Gastrosquisis.
27. ¿Cuál de las siguientes manifestaciones NO forma parte de la 5) Ombligo amniótico.
tétrada de Sabin?
32. Recién nacida, de 3 semanas de vida, que presenta valores de TSH
1) Coriorretinitis. de 75 μg/ml (0,3 –5 ) en el cribado neonatal de hipotiroidismo.
2) Cataratas. Con respecto a esta situación, es FALSO:
3) Crisis convulsivas.
4) Hidrocefalia. 1) Su causa más frecuente es la disgenesia tiroidea.
5) Calcificaciones cerebrales. 2) Situaciones de dismorfogénesis pueden asociarse a sordera
neurosensorial.
28. Un RN de 38 semanas de edad gestacional y 1.750 g, presenta 3) Un tratamiento precoz supone un mejor pronóstico neuro-
erupción purpuropetequial, hepatoesplenomegalia y se lógico.
ausculta un soplo continuo áspero en 2.º espacio intercostal 4) Los signos clínicos del hipotiroidismo están ya presentes
izquierdo. En la Rx de tórax se objetiva cardiomegalia con inmediatamente al nacimiento.
aumento de la vascularización pulmonar así como imágenes 5) En situaciones de mal control pueden acabar desencade-
lineales radiotransparentes con aumento de densidad ósea nando una pubertad precoz.
en la zona metafisaria del húmero sin reacción perióstica. En
relación con el cuadro que presenta este paciente, señale la 33. En relación con el desarrollo psicomotor en el niño, señale la
respuesta FALSA: respuesta FALSA:

1) Cuando la madre contrae la infección en el tercer trimestre 1) La sonrisa social referencial aparece ente el 1.º y 2.º mes.
aumenta el riesgo de infección del feto, pero el cuadro clínico 2) El sostén cefálico se inicia a los 3 meses.
es más leve. 3) La sedestación estable se consigue a los 6-8 meses.
2) Si la infección se produce después de la 12.ª semana de 4) Emite sus primeros fonemas con 3-4 semanas.
gestación, es poco probable la asociación de una cardiopatía 5) Realiza frases cortas con 2 años.
congénita.
3) La presencia de IgM específica antirrubéola refleja la pro- 34. En relación con el crecimiento y el desarrollo del niño en su
ducción en el útero de anticuerpos por el feto. primer año de vida, ¿cuál de las siguientes afirmaciones es
4) El lactante puede permanecer con infección crónica durante FALSA?
meses después del nacimiento.
5) El virus puede aislarse en sangre. 1) El niño aproximadamente duplica su peso al 5.º mes y lo
triplica al año de vida.
29. De los siguientes, el factor más relacionado con el crecimiento 2) El peso del RN puede disminuir un 5-10% en la primera
intrauterino retardado tipo simétrico es la: semana de vida por pérdida de líquido extravascular.
3) Los reflejos precoces o arcaicos presentes al nacimiento
1) Infección congénita. suelen desaparecer entre los 3 y 4 meses de edad.
2) Isoinmunización Rh. 4) Hacia los 2 meses suele haber hecho su aparición la sonrisa
3) Hipertensión. social.
4) Diabetes con vasculopatía. 5) Duplican la talla del nacimiento poco después de cumplir
5) Edad de la mujer por encima de 35 años. el primer año de edad.

CTO Medicina • C/Francisco Silvela, 106 • 28002 - Madrid • Tfno. (0034) 91 782 43 30/33/34 • E-mail: secretaria@ctomedicina.com • www. ctomedicina.com 4
Test 1.ª vuelta Distancia
Pediatría
35. Con respecto al calostro, es FALSO que: 4) Laringitis estridulosa.
5) Epiglotitis.
1) Se denomina así a la secreción mamaria durante los primeros
2-4 días postparto. 40. Ante un niño de 2 años de edad que, estando previamente bien,
2) Contiene más proteínas y más carbohidratos que la leche se despierta bruscamente por la noche con tos perruna, estridor
humana madura. y dificultad respiratoria, pulso acelerado y afebril, pensaremos
3) Su densidad es mayor que la de la leche humana madura. en:
4) Las características de leche humana madura se adquieren
alrededor de la 3.ª-4.ª semana. 1) Laringotraqueítis aguda.
5) Posee factores inmunológicos. 2) Laringitis espasmódica.
3) Epiglotitis aguda.
36. Respecto a las diferencias entre la leche materna y la leche de 4) Difteria.
vaca, señale la afirmación correcta: 5) Traqueítis bacteriana.

1) La leche de vaca tiene un contenido energético dos veces 41. Varón de 5 años que, estando previamente bien, despierta por la
superior al de la leche materna. noche con fiebre de 39,8 º C, voz apagada, intenso babeo, estridor
2) La leche de vaca contiene una cantidad de calcio superior inspiratorio con escasa tos. A la exploración destaca mal estado
a la leche materna. general, estridor inspiratorio e intenso tiraje supra-esternal y
3) La leche materna contiene mayor cantidad de vitamina K subcostal, así como tendencia a la hiperextensión del cuello. Se
que la leche de vaca. realiza radiografía lateral de faringe, observando ocupación de
4) La leche de vaca es más rica en hidratos de carbono que la espacio supraglótico. En el hemograma destaca una leucocitosis
leche materna. con desviación izquierda. ¿Qué medida terapéutica NO estaría
5) La leche materna tiene una relación caseína/seroproteínas aconsejada en este caso?
de 70/30.
1) Aplicar oxígeno e intentar obtener una vía aérea artificial
37. La estatura de un niño de 4 años está por debajo del percentil en quirófano.
3. Su velocidad de crecimiento es normal. La edad ósea es de 2) Iniciar tratamiento antibiótico con ceftriaxona.
2 años. El padre presentó un retraso en la pubertad, aunque 3) Aplicar adrenalina racémica y corticoides.
actualmente su talla está dentro de los límites normales. Este 4) Usar las medidas generales para descender la hipertermia
proceso se denomina: que sufre el paciente.
5) Mantener tranquilo al paciente.
1) Hipopituitarismo primario.
2) Hipopituitarismo secundario. 42. Lactante varón de 3 meses que desde hace dos días presenta
3) Retraso constitucional del crecimiento. cuadro de rinorrea serosa con estornudos y tos seca acompañado
4) Talla baja genética. de rechazo de las tomas. Hoy acude a Urgencias por cuadro de
5) Síndrome de Laron. dificultad respiratoria con taquipnea y tos sibilante paroxística.
Exploración física: 37,7 ºC, regular estado general, acianótico,
38. Acude a su consulta un padre con su hijo de 8 años preocupado con aleteo nasal y tiraje subcostal e intercostal. Auscultación pul-
porque dice que su hijo es el niño más bajo de toda su clase. monar: sibilancias espiratorias difusas con espiración alargada.
Su talla actual se encuentra en el percentil 2 y su talla genética Se realiza hemograma con 3,5 millones de hematíes; Hb de 10
está situada en el percentil 3. Al realizar el estudio de la edad g/dl; Hto de 32%; 7.000 leucocitos/mm3 con 50% neutrófilos,
ósea del niño se ve que ésta coincide con su edad cronológica. 35% linfocitos y 5% monocitos. En la Rx de tórax destaca hipe-
¿Qué cree usted que explicaría la talla baja de este niño? rinsuflación pulmonar. Señale la FALSA respecto a la evolución
clínica de esta enfermedad:
1) Un hipocrecimiento por deficiencia de la hormona de cre-
cimiento. 1) La fase más crítica son las primeras 48-72 horas desde el
2) Una alteración cromosómica. comienzo de la tos y la disnea.
3) Un hipotiroidismo congénito. 2) En menores de un mes puede cursar como apnea con escasez
4) Un retraso constitucional del crecimiento. de clínica respiratoria.
5) Una talla baja familiar. 3) Después de la fase crítica mejoran muy rápidamente.
4) La ribavirina podría usarse en lactantes con cardiopatías
39. Niña de 2 años que acude a Urgencias por cuadro de disnea, congénitas o displasia broncopulmonar.
estridor y tos perruna. En la exploración destaca rinorrea 5) Las complicaciones bacterianas como la bronconeumonía
intensa, amígdalas hipertróficas con exudado puntiforme. u OMA son frecuentes.
Fiebre de 38,5º C. A la auscultación pulmonar se aprecia hipo-
ventilación bilateral y estridor inspiratorio. El diagnóstico más 43. Paciente de 2 años de edad consulta por estancamiento ponde-
probable será: roestatural (< P3). Entre los antecedentes personales destacan
en el último año episodios repetidos de bronquiolitis con regular
1) Traqueítis bacteriana. recuperación posterior, persistiendo una tos crónica. Además
2) Laringitis supraglótica. presenta diarrea intermitente sin acompañarse de fiebre ni
3) Laringitis aguda. vómitos. ¿Cuál sería el diagnóstico más probable?

CTO Medicina • C/Francisco Silvela, 106 • 28002 - Madrid • Tfno. (0034) 91 782 43 30/33/34 • E-mail: secretaria@ctomedicina.com • www. ctomedicina.com 5
Test 1.ª vuelta Distancia
Pediatría
1) Enfermedad celíaca. 1) No suele haber antecedentes familiares de atopia.
2) Tuberculosis. 2) La proteína comúnmente implicada es la betalactoglobulina.
3) Asma. 3) Las pruebas cutáneas serán positivas.
4) Fibrosis quística. 4) El diagnóstico definitivo se realiza mediante un test de
5) Déficit de alfa-1 antitripsina. supresión-provocación.
5) Suele ser un proceso transitorio.
44. Sobre la traqueítis bacteriana, todo es cierto EXCEPTO:
49. RN de pocas horas de vida que presenta exceso de secreciones
1) La causa más frecuente es Haemophilus influenzae. orales, episodios de cianosis y atragantamiento durante las
2) Debe sospecharse ante todo crup viral con fiebre que no tomas junto con un abdomen muy distendido y timpánico a
responde al tratamiento. la percusión. Entre sus antecedentes destacan polihidramnios
3) No estarán presentes los hallazgos clínicos de una epiglotitis. materno durante la gestación e imposibilidad de pasarle una
4) Será necesario instaurar tratamiento antibiótico. sonda nasogástrica hasta el estómago. En relación con la pato-
5) En ocasiones puede requerir uso de vía aérea artificial. logía que sospecha, refiera cuál de las complicaciones siguientes
al tratamiento quirúrgico es la más frecuente:
45. Señale la respuesta FALSA, al hablar de la fibrosis quística:
1) Fístula anastomótica.
1) La lesión anatomopatológica pulmonar más precoz es la 2) Estenosis esofágica.
bronquiolitis. 3) Fístula traqueoesofágica recidivante.
2) Un 10% de los casos debuta con un íleo meconial. 4) Reflujo gastroesofágico.
3) La función endocrina del páncreas se altera con el tiempo. 5) Traqueomalacia.
4) La aspergilosis broncopulmonar alérgica es excepcional en
estos pacientes y se trata con antifúngicos como el vorico- 50. Varón de 3 meses que consulta por vómitos desde hace un mes.
nazol. Vomita después de todas las tomas, sin fuerza; come siempre
5) Pueden presentar deshidratación con alcalosis hipoclorémica con buen apetito y presenta adecuado desarrollo ponderoes-
en ambientes calurosos. tatural, sin pérdida de peso. Realiza dos deposiciones diarias
con heces de consistencia, aspecto y volumen normal. Entre sus
46. En el tratamiento de la fibrosis quística, NO es cierto que: antecedentes destaca estar alimentado con fórmula artificial
desde el mes de vida. Su actitud sería:
1) La fisioterapia respiratoria alarga la supervivencia.
2) Los productos enzimáticos micronizados disminuyen la 1) Cambiar su fórmula por un hidrolizado de proteínas de vaca.
esteatorrea. 2) Hacer endoscopia para descartar esofagitis.
3) Se necesita aportar suplementos de vitaminas liposolubles 3) Solicitar ecografía abdominal y/o tránsito digestivo.
a la dieta. 4) Realizar manometría esofágica.
4) La colonización de la vía respiratoria por P. aeruginosa es 5) Indicar a la madre medidas posturales y usar espesante de
fácilmente revertida con la antibioterapia adecuada. la leche.
5) En ocasiones son necesarios lavados y aspiración traqueo-
bronquial broncoscópica. 51. Señale la respuesta verdadera respecto a la estenosis hipertrófica
de píloro:
47. Varón de 13 meses que desde hace 2 días presenta fiebre de
38 ºC y síntomas catarrales. Hoy acude por presentar vómitos 1) Suele ocurrir en la primera semana de vida.
no biliosos y 6 deposiciones en las últimas 2 horas, líquidas, no 2) Es más frecuente en niñas.
fétidas, sin moco ni pus ni sangre. Respecto al cuadro que pre- 3) Presenta alcalosis metabólica hiperclorémica con hipopo-
senta este paciente, señale cuál de las siguientes afirmaciones tasemia.
es FALSA: 4) La prueba diagnóstica de elección es la Rx de abdomen.
5) El tratamiento consiste en la pilorotomía submucosa de
1) El agente causal más frecuente en la infancia es el rotavirus. Ramstedt.
2) Suele ser un proceso autolimitado, que cede en 3-10 días.
3) Puede causar deshidratación isotónica con acidosis. 52. RN con síndrome de Down presenta vómitos biliosos a las 36
4) Es frecuente que tienda a la cronificación. horas de vida. A la exploración destaca abdomen excavado. El
5) El diagnóstico puede hacerse mediante detección rápida diagnóstico más probable es:
de antígeno de rotavirus en heces.
1) Estenosis hipertrófica de píloro.
48. Lactante de 3 meses, alimentado al pecho hasta hace una semana 2) Enfermedad de Hirschprung.
en que, debido a una hipogalactia, se decide suplementar con 3) Divertículo de Meckel.
una fórmula artificial. Hoy acude a Urgencias por presentar 4) Atresia duodenal.
anorexia, vómitos, pérdida de peso así como exantema urti- 5) Invaginación intestinal.
carial en tronco. Entre sus antecedentes destaca: RNT, nacido
por cesárea y haber iniciado alimentación oral a las 4 horas de 53. Con respecto al divertículo de Meckel, es FALSO que:
vida con fórmula artificial. Con respecto a la patología actual,
es FALSO que: 1) Afecta al 1-2% de la población.

CTO Medicina • C/Francisco Silvela, 106 • 28002 - Madrid • Tfno. (0034) 91 782 43 30/33/34 • E-mail: secretaria@ctomedicina.com • www. ctomedicina.com 6
Test 1.ª vuelta Distancia
Pediatría
2) Se localiza en íleon a unos 75 cm de la válvula ileocecal. 57. Ante una enferma con clínica sospechosa de enfermedad celíaca,
3) Se denomina hernia de Littré cuando el divertículo de Meckel señale lo correcto respecto a su diagnóstico:
se aloja en una hernia inguinal indirecta.
4) Su manifestación clínica más frecuente es una hemorragia 1) Ante la sospecha muy alta con clínica florida, se debe excluir
gastrointestinal acompañada de dolor abdominal. el gluten de la dieta, sin necesidad de realizar biopsia.
5) El método diagnóstico más útil es la gammagrafía con 2) Los Acs antitransglutaminasa son los más específicos.
Tc-99. 3) La anatomía patológica de la muestra de biopsia es patog-
nomónica.
54. Un varón de 14 meses, previamente sano, de forma brusca 4) Antes de realizar la primera biopsia, es necesario excluir el
presenta crisis de llanto con encogimiento de piernas y palidez, gluten.
separados por períodos asintomáticos de duración variable en 5) El control de la enfermedad se basa en la realización de
los que permanece decaído. Ha presentado dos vómitos y una biopsias anuales.
deposición semilíquida sanguinolenta. Exploración física: afebril;
palpación de una masa alargada y mal definida, localizada en 58. Neonato varón, de 3 semanas de vida, acude a su consulta por
hipocondrio derecho. En la Rx de abdomen aparece silencio ictericia con coluria e hipocolia. A la exploración se palpa hepa-
aéreo en hipocondrio derecho. Señale cuál de las siguientes tomegalia. En la gammagrafia con HIDA, la captación es normal,
afirmaciones es cierta respecto al cuadro que presenta este pero la excreción es nula. Sobre la entidad que sospecha, señale
paciente: la opción FALSA:

1) Debe retrasarse el tratamiento durante 4-6 horas debido a 1) Se puede asociar a poliesplenia.
la alta incidencia de reducción espontánea. 2) El tratamiento definitivo es la hepatoportoenterostomía.
2) La recidiva es menos frecuente después de la reducción 3) En la biopsia es posible hallar células gigantes.
hidrostática que tras la reducción quirúrgica. 4) Es infrecuente la aparición de varios casos en una misma
3) Si hay signos de obstrucción intestinal de 48 horas de evo- familia.
lución, es preferible la reducción quirúrgica. 5) En el sondaje duodenal se suele observar ausencia de bilis.
4) Una invaginación asociada a divertículo de Meckel suele
reducirse satisfactoriamente con enema opaco. 59. Paciente de 5 meses que acude por presentar, desde hace 2 días,
5) La mortalidad es muy elevada, aunque la reducción tenga hasta 10 deposiciones al día, líquidas sin moco ni sangre, así
lugar en las primeras 24 horas. como dolor medio abdominal sin relación temporal. Entre sus
antecedentes destaca diarrea sanguinolenta hace una semana.
55. Niño de 4 meses que es llevado a consulta por estreñimiento En la exploración destaca: afebril, buen estado general, disten-
crónico desde el nacimiento. Entre sus antecedentes personales sión abdominal con abundantes ruidos hidroaéreos y eritema
destaca tapón meconial al nacimiento que se resolvió con en región del pañal. En el examen macroscópico de heces se
enemas. Alimentación materna exclusiva. Talla y peso < P3. detecta presencia de azúcares reductores. Este cuadro es:
¿Cuál de los siguientes datos NO apoyaría el diagnóstico?
1) Déficit de sacarasa-isomaltasa.
1) Biopsia rectal patológica. 2) Déficit aislado de isomaltasa.
2) Heces acintadas. 3) Déficit de lactasa.
3) Ampolla sin heces en el tacto rectal. 4) Déficit de enteroquinasa.
4) Relajación del esfínter anal interno. 5) Gastroenteritis por rotavirus.
5) Permanencia del bario en colon durante más de 24 horas al
realizar un tránsito digestivo. 60. Lactante de un mes de edad, nacido tras un embarazo controlado
y normal, a las 35 semanas de edad gestacional, que acude a
56. Paciente de 12 meses que, de forma progresiva, inicia un cuadro su consulta para la “revisión del niño sano” correspondiente.
de inapetencia, mal carácter, retraso del crecimiento ponde- Todo parece estar bien salvo que los testículos se encuentran
roestatural, así como diarrea abundante, fétida y de aspecto en el canal inguinal, y aunque sí lo hacen con tracción manual,
grasiento. Exploración física: afebril, regular estado general, espontámente no bajan al escroto. ¿Cuál es la actitud más ade-
palidez cutaneomucosa, pérdida de masa muscular proximal y cuada en este momento?
abdomen distendido. Señale lo FALSO respecto a la enfermedad
celíaca: 1) Si la alteración es bilateral, hay que consultar con el cirujano
cuanto antes, para que realice una orquidopexia, por el riesgo
1) Existe una predisposición genética, relacionada con los HLA de esterilidad y degeneración maligna.
B8, DR7, DR3 y DQ2. 2) Hay que esperar, porque es posible que espontánemente
2) Hay una mayor prevalencia de la enfermedad en niños con se complete el descenso testicular.
déficit selectivo de IgA o diabetes mellitus. 3) Trataría con HCG vía intramuscular a días alternos, y si no
3) El período más común de presentación es entre los 6 meses hay respuesta, mandaría al cirujano para la extirpación de
y los 2 años de edad. ambos testes.
4) Los anticuerpos antiendomisio IgA tienen una alta especi- 4) Se trata de unos testículos en ascensor, lo que no requiere
ficidad y sensibilidad. ningún tipo de tratamiento ni seguimiento.
5) El trigo, arroz, cebada y centeno deben ser excluidos de la 5) Trataría con testosterona y HCG, y si no hay respuesta, con-
dieta para toda la vida. sultaría con los cirujanos para su descenso.

CTO Medicina • C/Francisco Silvela, 106 • 28002 - Madrid • Tfno. (0034) 91 782 43 30/33/34 • E-mail: secretaria@ctomedicina.com • www. ctomedicina.com 7
Test 1.ª vuelta Distancia
Pediatría
61. Lactante de 3 meses consulta por cuadro de vómitos y febrícula tiempos de protrombina y de tromboplastina parcial activada
de 48 horas de evolución, con progresiva pérdida del apetito. son normales. Ante este cuadro clínico, una de las siguientes
En la exploración no se objetiva foco infeccioso. Hemograma afirmaciones es cierta:
con leucocitosis y desviación a la izquierda. Sistemático de
orina: leucocitos y nitritos positivos. ¿Cuál sería la actitud más 1) El diagnóstico más probable es el de púrpura trombótica
adecuada? trombocitopénica.
2) Requiere de transfusión urgente de concentración de pla-
1) Tratamiento con antitérmicos y ver evolución. quetas.
2) Realizar punción lumbar. 3) La evolución más probable es a la recuperación espontánea.
3) Diagnóstico de infección de orina y administrar antibióticos 4) Debe practicarse una TC craneal para descartar hemorragia
v.o. intracraneal.
4) Realizar punción suprapúbica e iniciar antibióticos. 5) El pronóstico depende de la precocidad del tratamiento.
5) Descartar reflujo gastroesofágico.
66. Los tumores malignos son la principal causa de muerte entre
62. Es FALSO, respecto al reflujo vesicoureteral en el niño: el año y los 15 años de edad en los países desarrollados.
Señale la afirmación INCORRECTA respecto a su epidemio-
1) Puede sospecharse intraútero al ver dilatación pielocalicial logía:
en la ecografía obstétrica.
2) Suele dar lugar a infecciones urinarias de repetición. 1) La leucemia linfoblástica aguda es la neoplasia maligna más
3) La indicación quirúrgica parte del número anual de recidivas frecuente en la infancia, y muestra un ligero predominio en
de infección urinaria. varones.
4) Puede ser útil hacer profilaxis antibiótica si se han repetido 2) La incidencia más alta de enfermedad de Hodgkin se da en
varias infecciones urinarias. adolescentes y adultos jóvenes.
5) Si el reflujo es masivo, puede llevar a IRC en la infancia y 3) El neuroblastoma es el tumor sólido más frecuente en la
puede requerir cirugía. infancia, fuera del sistema nervioso central.
4) El sarcoma de Ewing es el tumor óseo primario maligno
63. Varón de 2 años en cuya historia clínica destaca haber padecido más frecuente a partir de los 10 años, seguido del osteo-
un proceso diarreico en los días previos. En la exploración se sarcoma.
observa palidez, estupor y hematuria. Ha presentado además 5) El rabdomiosarcoma es el sarcoma de tejidos blandos más
una convulsión focal. Respecto al cuadro que usted sospecha, frecuente en la infancia, siendo la cabeza y el cuello las
señale cuál de las siguientes opciones es FALSA: principales localizaciones.

1) Es la causa más frecuente de IRA en los niños pequeños. 67. Niña de 2 años presenta movimientos mioclónicos de extremi-
2) Existe anemia hemolítica microangiopática y trombopenia dades y sacudidas desordenadas de los ojos. En la exploración
dentro del cuadro clínico. se palpa masa abdominal en línea media y flanco derecho. Se le
3) La trombosis bilateral de las venas renales puede provocar realiza una TC abdominal, evidenciándose una masa de consis-
un cuadro similar. tencia mixta y calcificaciones en su interior. Señale la afirmación
4) El empleo de corticoides supone el tratamiento de elección. falsa:
5) La aparición de recidivas es muy poco frecuente.
1) Es el tumor extracraneal más frecuente en la infancia.
64. Varón de 5 años que presenta de forma súbita dolor en teste 2) La edad más habitual al diagnóstico son los 2 años.
derecho, sin antecedente de traumatismo previo. Exploración: 3) La presentación con un síndrome de mioclonus- opsoclonus
afebril, teste tumefacto, doloroso a la exploración, con ausencia ensombrece el pronóstico.
del reflejo cremastérico. Señale la afirmación FALSA: 4) Las catecolaminas en orina estarán elevadas.
5) Se asocia con deleción del cromosoma 1 y anomalías del 17.
1) En la eco-Doppler se observaría disminución del flujo san-
guíneo en ese testículo. 68. Señale la opción FALSA respecto al tumor de Wilms:
2) El tratamiento consiste en antibióticos, antiinflamatorios, y
tras la fase aguda, cirugía. 1) Se asocia a deleción del cromosoma 11, ya sea en las células
3) La mayor incidencia se produce en la niñez tardía y adoles- tumorales o en todas las células del organismo.
cencia temprana. 2) Se asocia a aniridia y anomalías genitourinarias.
4) En mayores de 13 años se debe hacer diagnóstico diferencial 3) La HTA es un hallazgo frecuente.
con una epididimitis. 4) El diagnóstico definitivo nos lo da la biopsia.
5) En el periodo neonatal, generalmente tiene mal pronóstico. 5) Suele tratarse de una masa abdominal palpable asintomática,
con o sin metástasis pulmonares.
65. Un niño de 2 años es traído a Urgencias por petequias generali-
zadas sin otros signos de diátesis hemorrágica. Refieren catarro 69. Un niño de 5 años, previamente sano, debuta bruscamente con
de vías altas y fiebre 2 semanas antes pero en la actualidad palidez, astenia y anorexia; se le practica un hemograma y se
de encuentra afebril, con buen estado general, y el resto de la evidencia anemia, leucopenia y trombopenia. En la exploración
exploración es normal. Un hemograma muestra hemoglobina se aprecia hepatoesplenomegalia. ¿Qué diagnóstico le parece
14 mg/dl, leucocitos 9.400/mm3 y plaquetas 34.000 mm3. Los más probable?

CTO Medicina • C/Francisco Silvela, 106 • 28002 - Madrid • Tfno. (0034) 91 782 43 30/33/34 • E-mail: secretaria@ctomedicina.com • www. ctomedicina.com 8
Test 1.ª vuelta Distancia
Pediatría
1) Anemia ferropénica. plaquetas 71.000/ mm3. De momento no dispone de frotis de
2) Linfoma no Hodgkin. sangre periférica, ni de reactantes de fase aguda, pero ya tiene
3) Aplasia medular adquirida. una sospecha diagnóstica. Señale cuál es:
4) Aplasia medular congénita.
5) Leucemia aguda. 1) Mononucleosis infecciosa.
2) Leucemia linfoblástica aguda.
70. Con respecto al tumor de Wilms, es FALSO que: 3) Linfoma en fase leucémica.
4) Hemoglobinuria paroxística nocturna (HPN).
1) Puede cursar con poliglobulia. 5) Aplasia medular por antibióticos.
2) En la UIV se observa una masa dependiente del tejido renal.
3) La cirugía se indica, aun en presencia de metástasis pulmo- 74. Un niño de 4 años acude al pediatra por aparición de lesiones
nares. papulosas rojas, algunas con vesículas blanquecinas no umbi-
4) Posee gran tendencia a presentar calcificaciones intraneoplá- licadas en tronco y mucosa oral, muy pruriginosas. Dos días
sicas. antes presentaba cuadro catarral con fiebre moderada. Respecto
5) Los tumores bilaterales son más frecuentes en los casos al cuadro que presenta este niño, todo lo siguiente es cierto,
familiares. EXCEPTO:

71. Niña de 6 años que presenta desde hace 3 días fiebre de 38 ºC 1) Está causado por un poxvirus, el virus de la varicela zóster.
que no cede con antitérmicos, tos seca y facies congestiva. Hoy 2) Las lesiones costrosas no contienen virus viables.
acude por aparecer exantema maculopapuloso, rojo intenso, 3) La complicación más frecuente es la sobreinfección de las
no puntiforme, confluente, que no se blanquea a la presión lesiones cutáneas.
en la parte superior del tórax, cara y parte proximal de EE.SS. 4) La neumonía varicelosa es poco frecuente en la infancia.
A la exploración: Tª 40,3 ºC, adenopatías bilaterales en ángulo 5) La encefalitis postvaricela que se presenta con signos cere-
mandibular y lesiones blanquecinas sobre base eritematosa en belosos tiene mejor pronóstico que si lo hace con signos
mucosa subyugal. Respecto a la enfermedad que padece esta cerebrales.
paciente, señale la afirmación FALSA:
75. Niño de 7 años presenta rash eritematoso en ambas mejillas,
1) El exantema comienza por la cara, tiene evolución descen- tronco y zona proximal de extremidades, con tendencia a acla-
dente, y desaparece en el mismo orden en el que apareció. rarse en la zona central. Con respecto a esta enfermedad, señale
2) La neumonía de células gigantes de Hecht es menos fre- la afirmación FALSA:
cuente que la neumonía por sobreinfección bacteriana.
3) La gravedad de la enfermedad está directamente relacionada 1) Es producida por el parvovirus B19.
con la intensidad y confluencia del exantema. 2) El período de incubación es de 7 días.
4) El prurito suele ser intenso. 3) El período prodrómico se caracteriza por fiebre alta.
5) Entre las posibles complicaciones se encuentra la anergia 4) El exantema característico aparece en tres etapas.
cutánea y la reactivación de una TBC preexistente. 5) Como complicación puede producirse una crisis aplásica
grave.
72. Niña de 5 años presenta cuadro catarral con fiebre de 37,5
ºC. A la exploración presenta adenopatías retroauriculares y 76. Un lactante de 7 meses presenta súbitamente fiebre de 40 ºC.
cervicales dolorosas a la palpación e hiperemia conjuntival. No existe ningún otro síntoma y la exploración física es normal,
48 horas después, aparece un exantema morbiliforme en cara. salvo el hallazgo de una coriza ligera y adenopatías cervicales
¿Cuál de las siguientes afirmaciones sobre la epidemiología de posteriores. El niño parece encontrarse bastante bien. ¿Cuál de
esta enfermedad es cierta? las siguientes afirmaciones describe mejor los factores diagnós-
ticos que deben considerarse en este caso?
1) Las personas con enfermedad subclínica no son contagiosas.
2) Los niños se afectan con mayor frecuencia que las niñas. 1) Si apareciera una erupción cutánea 24 horas después de
3) Los anticuerpos maternos protegen los 6 primeros meses comenzar la fiebre, sería probable el diagnóstico de roséola
de vida. o exantema súbito.
4) Un solo ataque no suele conferir inmunidad de duración 2) En la roséola infantil, el diagnóstico diferencial puede ser
permanente. difícil, puesto que en las primeras 36 horas cursa con leu-
5) La relación de enfermedad no aparente/manifiesta es de 1 cocitosis, con predominio de neutrófilos.
a 1. 3) La fiebre podría corresponder a los pródromos de una
rubéola.
73. Niño de 11 años, que consulta por fiebre, dolor de garganta 4) Si la fiebre dura 3 días, desaparece luego rápidamente y
y cansancio, de 5 días de evolución. En la exploración física aparece una erupción cutánea en ese momento, sería pro-
destacan unas amígdalas hipertróficas con exudado en sábana, bable el diagnóstico de eritema infeccioso.
adenopatías dolorosas axilares, inguinales, y las más grandes, 5) Es probable una escarlatina, pues es una enfermedad propia
las cervicales, y hepatoesplenomegalia de 3-4 cm por debajo de niños de 6 a 18 meses.
de los rebordes costales. Pide un análisis de sangre, y lo pri-
mero que le mandan del laboratorio es el hemograma: Hb 8,1 77. Jennifer tiene 16 meses de edad y acude a Urgencias por pre-
g/dl; Hto 27%; leucocitos totales 1.800/ mm3 (600 neutrófilos); sentar desde hace 2 días fiebre de 39 ºC que su pediatra atribuyó

CTO Medicina • C/Francisco Silvela, 106 • 28002 - Madrid • Tfno. (0034) 91 782 43 30/33/34 • E-mail: secretaria@ctomedicina.com • www. ctomedicina.com 9
Test 1.ª vuelta Distancia
Pediatría
a cuadro respiratorio. A la exploración presenta fiebre de 40 3) La neumonía por MAI es la infección oportunista más fre-
ºC y contractura antiálgica en flexión de extremidad inferior cuente en el SIDA pediátrico.
izquierda. Queda ingresada para estudio, y 4 días más tarde 4) La manifestación más frecuente en el SIDA infantil es la
persiste el cuadro febril, junto con edema indurado en ambos hepatitis B.
pies, fisuras labiales, conjuntivitis no purulenta y exantema 5) La parotiditis es más frecuente en el SIDA infantil que en el
polimorfo en tronco. El diagnóstico más probable será: del adulto.

1) Rubéola. 82. Sobre el diagnóstico de un niño VIH positivo, es FALSO que:


2) Mononucleosis infecciosa.
3) Púrpura de Schönlein-Henoch. 1) Una IgG+ en un lactante de 10 meses indica infección activa
4) Enfermedad de Kawasaki. por el VIH.
5) Eritema infeccioso. 2) La mayoría de los niños infectados presenta hipergamma-
globulinemia policlonal precoz.
78. Paciente varón de 5 años de edad que acude por presentar 3) La linfopenia e inversión del cociente CD4/CD8 son menos
cuadro febril y odinofagia con exudado blanquecino en ambas llamativas que en el adulto.
amígdalas que su pediatra trató con penicilina oral. Dos días 4) No es útil la determinación de IgA contra el VIH, porque no
después, presenta petequias y púrpura palpables, principal- suele aparecer hasta el 6.º mes de vida.
mente en miembros inferiores, así como artralgias en ambos 5) Los resultados positivos en la PCR han mostrado una corre-
tobillos que le impiden la marcha y dolor abdominal de tipo lación positiva con el aislamiento del virus en cultivo.
cólico. Los estudios complementarios revelan 13 g/dl de Hb;
leucocitos 10.500/mm3 con fórmula normal; plaquetas 485.000/ 83. Respecto a la vacuna triple vírica, señale la afirmación FALSA:
mm3; tiempo de protrombina de 95% y tiempo parcial de trom-
boplastina de 27 segundos (control 25 segundos). La causa más 1) Es una vacuna de microorganismos atenuados.
probable de estos hallazgos es: 2) La alergia al huevo ha dejado de ser una contraindicación
absoluta para su administración.
1) Reacción a la penicilina. 3) Está contraindicada su utilización en embarazadas y
2) Púrpura de Schönlein-Henoch. pacientes VIH positivos.
3) Mononucleosis infecciosa. 4) Su administración en el calendario actual se realiza a los 15
4) Enfermedad de Kawasaki. meses y a los 4 años.
5) Púrpura trombocitopénica idiopática. 5) En zonas de elevada morbimortalidad puede administrarse
la primera dosis a los 6-12 meses, con una segunda dosis de
79. Respecto a la mononucleosis infecciosa, es FALSO que: refuerzo después del año de vida.

1) Hasta en el 80% hay aumento de transaminasas. 84. Señale la afirmación FALSA respecto a la vacuna de la polio:
2) En la analítica aparece leucopenia.
3) Títulos elevados de anticuerpos frente al virus de Epstein- 1) La vacuna tipo Sabin es de virus vivos atenuados.
Barr. 2) La vacuna oral produce inmunidad local (IgA) y general (IgG).
4) Existe un síndrome linfoproliferativo en varones que han 3) En el calendario actual se administra a los 2-4-6-18 meses y
sufrido la infección por este virus. entre los 4 y 6 años.
5) La complicación más temida es la rotura esplénica. 4) La vacuna tipo Salk está contraindicada en familiares de
inmunodeprimidos.
80. Un bebé de 3 meses, que recibe lactancia materna, presenta una 5) En caso de diarrea o vómitos sería conveniente retrasar la
historia de accesos de tos con congestión facial, lagrimeo y acaba vacunación con virus atenuados.
vomitando. Hace 2 semanas presentó cuadro catarral de vías áreas
altas. En la Rx tórax se aprecian breves infiltrados perihiliares. Con 85. Señale la afirmación cierta respecto a la vacuna DTP:
respecto a este cuadro, señale la respuesta que NO sea cierta:
1) La vacuna de la tos ferina está contraindicada en mayores
1) Puede aparecer al principio de la vida por falta de anticuerpos de 18 meses.
maternos. 2) No puede administrarse conjuntamente con otras vacunas.
2) La fase catarral precede al período paroxístico. 3) La vacuna DTPa (acelular) tiene mayor riesgo de efectos
3) La tos paroxística puede durar de 1 a 4 semanas. secundarios que la DTP, por lo que no se utiliza de forma
4) El recuento de leucocitos suele elevarse en forma conside- rutinaria.
rable con predominio de polimorfonucleares. 4) A los 14 años es preciso vacunar con Td (tétanos y difteria
5) El período de incubación oscila entre 5 y 10 días. adultos), y no son necesarias nuevas dosis de recuerdo.
5) No está contraindicada en pacientes inmunodeprimidos.
81. Con respecto al SIDA en la infancia, señale la opción correcta:
86. Con respecto a la vacuna contra Haemophilus influenzae tipo B,
1) El SIDA precoz es menos frecuente que el tardío y la clínica es FALSO que:
predominante es infecciosa.
2) El SIDA tardío es más frecuente y predomina la clínica neu- 1) Las nuevas vacunas conjugadas son aptas para la inmuni-
rológica. zación en lactantes mayores de 2 meses.

CTO Medicina • C/Francisco Silvela, 106 • 28002 - Madrid • Tfno. (0034) 91 782 43 30/33/34 • E-mail: secretaria@ctomedicina.com • www. ctomedicina.com 10
Test 1.ª vuelta Distancia
Pediatría
2) Es muy inmunógena. 92. Ante un recién nacido de 2 días con cianosis, taquipnea sin
3) Sus efectos adversos son mínimos. soplo a la auscultación y que, en la radiografía de tórax,
4) Es necesario aplicarla después de los 5 años. muestra ligera cardiomegalia, con un pedículo cardíaco
5) En el calendario vacunal debería administrarse a los 2-4-6 y estrecho y aumento del flujo sanguíneo pulmonar, el diag-
18 meses. nóstico más probable es:

87. Señale la afirmación FALSA respecto a la vacuna del VHB: 1) Tetralogía de Fallot.
2) Coartación de aorta.
1) La vacuna de VHB está realizada por ingeniería genética con 3) Anomalía de Ebstein.
efectos adversos mínimos. 4) Transposición simple de las grandes arterias.
2) La pauta de administración en caso de riesgo inmediato 5) Atresia tricuspídea.
como contacto con una jeringuilla sería 0-1-2 meses, y al
año. 93. Lactante de 9 meses de edad que llega a Urgencias traído por
3) La vacuna VHB no está recomendada en mujeres embara- su madre, por somnolencia de 12 horas de evolución. No ha
zadas. tenido fiebre, vómitos, ni ninguna otra sintomatología. En la
4) No está indicada su administración en niños mayores de 7 actitud de la madre destaca que parece darle poca importancia
años. al estado del niño, contestando a las preguntas de la anam-
5) Se puede administrar con la gammaglobulina anti-VHB, nesis con desgana, aunque espontáneamente expresa todas
siempre que se administre en distinto punto. las dificultades por las que está pasando para compaginar su
papel de madre con su trabajo. En la exploración física, el niño
88. ¿Cuál sería la actitud más adecuada a seguir en los contactos está consciente y reacciona con irritabilidad a estímulos, pero
de un niño con meningitis meningocócica? tiende a quedarse dormido. Tiene un hematoma periorbitario
reciente, y varios pequeños antiguos en la región dorsolumbar,
1) Realizar quimioprofilaxis con ceftriaxona durante 2 días. que la madre atañe a caídas accidentales, aunque el niño está
2) No existe quimioprofilaxis eficaz. iniciando ahora la bipedestación con ayuda. ¿Qué sospecharía
3) Administración de la vacuna antimeningocócica en niños inicialmente como causante del estado del niño?
menores de 5 años.
4) Administración de rifampicina durante 2 días. 1) Meningitis bacteriana.
5) Administración de rifampicina durante 4 días. 2) Malos tratos.
3) Enfermedad metabólica no diagnosticada descompensada.
89. ¿Cuál sería la actitud a seguir ante un niño de 4 años asintomático 4) Hipoglucemia.
con Mantoux negativo, hijo de un tuberculoso bacilífero? 5) Hiponatremia.

1) Observación y repetir Mantoux en 6 meses. 94. Con respecto a la coartación de aorta (CoA), señale la FALSA:
2) Realizar quimioprofilaxis primaria durante 2 meses con INH
y repetir el Mantoux posteriormente. 1) El tipo más frecuente es la coartación de localización yux-
3) Realizar quimioprofilaxis primaria durante 6 meses con taductal.
INH. 2) La mayor parte de los casos se muestran asintomáticos.
4) Administrar tratamiento completo con tres fármacos durante 3) La tensión arterial en los miembros inferiores es mayor que
6 meses. en los superiores.
5) Vacunación con BCG. 4) En ocasiones, la tensión arterial en el brazo derecho es mayor
que en el izquierdo.
90. Respecto al síndrome de muerte súbita infantil o del lactante 5) En la radiografía de tórax, con el tiempo, se visualizan esco-
(SMSL), señale cuál de los siguientes NO constituye un factor taduras en los bordes inferiores de las costillas.
de riesgo:
95. Con respecto al desarrollo puberal normal, es verdadero:
1) Antecedentes familiares.
2) Prematuridad. 1) El primer dato a la exploración sugestivo de inicio de la
3) Lactancia materna. pubertad en las niñas es la pubarquia.
4) El sexo masculino. 2) El aumento del tamaño testicular en el varón es el primer
5) El tabaquismo materno. signo de pubertad.
3) La pubertad suele iniciarse a los 8 años en las niñas y a los
91. La persistencia del ductus arterioso se asocia a todo lo siguiente, 9 en los niños.
EXCEPTO a: 4) El vello sexual es el primer signo de pubertad.
5) La ganancia de peso y talla es más acusada en las niñas que
1) Pulsos periféricos saltones. en los niños.
2) Mayor frecuencia en prematuros.
3) Soplo continuo en el área pulmonar. 96. Niña de 4 años que consulta por desarrollo mamario lentamente
4) Puede ser efectivo el tratamiento con indometacina. progresivo desde hace 2 meses, sin adrenarquia ni aumento de la
5) Las prostaglandinas son el mejor tratamiento médico para velocidad de crecimento. Los padres no lo relacionan con ningún
conseguir su cierre. desencadenante ni con otros signos-síntomas. La exploración

CTO Medicina • C/Francisco Silvela, 106 • 28002 - Madrid • Tfno. (0034) 91 782 43 30/33/34 • E-mail: secretaria@ctomedicina.com • www. ctomedicina.com 11
Test 1.ª vuelta Distancia
Pediatría
física general es normal, con un estadio puberal II de Tanner (T2
P1 Aa). Se realiza una radiografía de mano y muñeca izquierdas,
estimándose la edad ósea en 3,5-4 años. En la ecografía pélvica
se visualizan útero y ovarios de morfología y tamaño adecuados
a la edad de la paciente, sin quistes significativos. Los niveles
séricos de estradiol son de 40 pg/dl (N 10-30), y la FSH y la LH
basales son normales. ¿Cuál es su diagnóstico de sospecha?

1) Pubertad precoz central, probablemente idiopática.


2) Pubertad precoz central, probablemente por hamartoma
hipotalámico.
3) Telarquia prematura aislada.
4) Pubertad precoz periférica.
5) Aunque no sepamos el índice de masa corporal (IMC) de la
paciente, lo más probable es que sea elevado y nos encon-
tremos realmente ante una adipomastia bilateral.

97. ¿Cuál de los siguientes cuadros no forma parte de diagnóstico


diferencial de talla baja en los niños?

1) Síndrome de Turner.
2) Síndrome de Down
3) Síndrome de Silver-Russell.
4) Síndrome de Klinefelter.
5) Síndrome de Noonan.

98. Respecto a las convulsiones neonatales, señale la afirmación


FALSA:

1) Los RN con convulsiones suelen tener mal pronóstico.


2) El tratamiento consiste en medidas de sostén y fenobarbital
o diazepam.
3) Las crisis tónico-clónicas generalizadas son raras durante el
primer mes de vida.
4) La clínica es variable y el EEG puede ser la única forma para
su diagnóstico.
5) La causa más frecuente de crisis en el período neonatal es
la encefalopatía hipoxicoisquémica.

99. Acerca de la enuresis, señale cuál es la respuesta correcta:

1) Es más frecuente en el sexo femenino.


2) La enuresis secundaria es más frecuente entre los 5 y los 8
años de edad, y tiene peor pronóstico que la primaria.
3) La predisposición genética es un factor de riesgo muy
importante para la enuresis nocturna.
4) La causa más frecuente de enuresis diurna es la infección
de orina.
5) El tratamiento de elección en la enuresis nocturna persistente
es la desmopresina administrada antes de acostarse.

100. Llega a Urgencias un niño de 18 meses con fiebre de 39 ºC, que


ha presentado en su domicilio un episodio tónico-clónico gene-
ralizado de 3 minutos de duración, con posterior somnolencia.
Lo correcto a su llegada a Urgencias sería:

1) Administración de bolo de glucosa.


2) Administración de antitérmicos y diazepam.
3) Administración de fenitoína.
4) Realizar punción lumbar.
5) Administración de piridoxina.

CTO Medicina • C/Francisco Silvela, 106 • 28002 - Madrid • Tfno. (0034) 91 782 43 30/33/34 • E-mail: secretaria@ctomedicina.com • www. ctomedicina.com 12
Comentarios de Test a distancia 1.ª vuelta
Pediatría

Comentarios de Test a distancia 1.ª vuelta

Pediatría
Pregunta 1.- R: 3 La anemia fisiológica, aparece antes y con más intensidad que en los
El test de Apgar se debe realizar a todo recién nacido, independientemente nacidos a término, por los menores depósitos de hierro y el rápido
de su edad gestacional sano o con sospecha de patología (respuesta 1 crecimiento, por lo que a veces precisan ser tratados con eritropoyeti-
falsa). Se realiza al minuto y a los 5 minutos y posteriormente sólo se na, que debe acompañarse siempre de suplementos de hierro vía oral
repetirá cada 5 minutos si la puntuación a los 5 minutos es inferior de (respuesta 2 correcta).
7 (respuesta 3 correcta). Las pérdidas insensibles de líquido están aumentadas respecto a los
Entre los parámetros que valora se encuentran: color, frecuencia cardíaca, neonatos nacidos a término, en relación inversa a la edad gestacional,
respuesta a la estimulación, tono muscular y esfuerzo respiratorio. NO y sus necesidades diarias son mayores (respuesta 3 correcta).
frecuencia respiratoria (respuesta 2 falsa). En todos los recién nacidos, pero más en los prematuros, los tóxicos
A pesar de una puntuación baja en los primeros minutos de vida, no de eliminación renal, y en los prematuros también los de eliminación
sirve para establecer valoraciones pronósticas (respuesta 4 falsa). Una hepática, deben darse en menor dosis o con intervalos mayores (res-
puntuación inferior a 3, mantenida más allá de los 20 minutos de vida, puesta 4 correcta).
sí puede predecir una elevada morbimortalidad. En lo RNPT, la hipoxia produce afectación de la matriz germinal, estructura
La palidez cutánea generalizada supone una puntuación de 0 en el test transitoria presente en el cerebro inmaduro hasta la semana 34 de gesta-
de Apgar (respuesta 5 falsa). ción. Dado que está compuesta por vasos su lesión produce hemorragia.
Hay que realizar sistemáticamente ecografía craneal a todos los recién
Pregunta 2.- R: 4 nacidos < 32 SG y a los recién nacidos > 32SG con factores de riesgo como
Si seguimos la tabla del comentario de la pregunta anterior obtendremos asfixia perinatal o signos neurológicos anormales (respuesta 5 correcta).
una puntuación de 4 en el test de Apgar realizado a este recién nacido.
Pregunta 4.- R: 3
Pregunta 3.- R: 1 La respuesta 1 es cierta: es importante para el MIR tener claro las diferencias
La alimentación de los prematuros debe realizarse de forma cautelosa. entre caput (edema de partes blandas, no limitado por suturas) y cefa-
Siempre que no haya distrés respiratorio ni otras circunstancias que lohematoma (hematoma entre hueso y periostio, limitada por suturas).
contraindiquen su inicio, hay que procurar una alimentación precoz La respuesta 2 es cierta: el traumatismo obstétrico más frecuente es la
para evitar la hipoglucemia, la deshidratación y la hiperbilirrubinemia. Se fractura de clavícula. A la exploración lo más llamativo es la crepitación
puede intentar vía oral y continuar si hay buena tolerancia, aunque en los y el reflejo de Moro asimétrico.
menores de 32 semanas es muy frecuente que no exista la coordinación La respuesta 3 es falsa: el llamado nódulo de Stromayer es un hematoma
adecuada y haya que colocar una SNG (respuesta 1 falsa). palpable a nivel del ECM. La mayoría de las veces se reabsorbe espontá-

0 1 2

Esfuerzo respiratorio Ausente Lento, irregular Bueno, llanto

Respuesta
a la introducción Sin respuesta Mueca Tos o estornudo
de una sonda nasogástrica

Frecuencia cardíaca Ausente Menos de 100 Más de 100

Tono Ligera flexión


Débil Movimientos activos
muscular de extremidades

Cuerpo sonrosado,
Color Azul, pálido Totalmente sonrosado
extremidades cianóticas

Pregunta 1. Test de Apgar.

CTO Medicina • C/Francisco Silvela, 106 • 28002 - Madrid • Tfno. (0034) 91 782 43 30/33/34 • E-mail: secretaria@ctomedicina.com • www. ctomedicina.com 1
Comentarios de Test a distancia 1.ª vuelta
Pediatría
neamente, pero en ocasiones se puede fibrosar acortando el músculo. La PCF suele cursar con clínica marcada de cianosis que no responde a la admi-
El tratamiento se basa en la fisioterapia y muy rara vez requiere cirugía. nistración de oxígeno. No hay una Rx de tórax característica (respuesta 4 falsa).
La respuesta 4 es cierta: la víscera que con mayor frecuencia se lesiona es La DBP es típica de pretérminos que han necesitado oxígeno con el
el hígado, pero la mayoría de las veces son pequeños hematomas sub- antecedente de barotrauma o EMH. Rx de tórax: patrón “en esponja”.
capsulares limitados que, secundariamente, pueden producir ictericia (respuesta 5 falsa).
precoz prolongada, sin otra repercusión clínica. Recordar para el MIR el
cuadro clínico típico de la hemorragia suprarrenal: niño grande, parto de Pregunta 7.- R: 2
nalgas, que en las primeras horas de vida presenta clínica de hipotensión, Estamos ante un cuadro típico de enfermedad de membrana hialina: RNPT
sangrado y shock. con dificultad respiratoria intensa que aparece en las primeras horas de
La respuesta 5 es cierta: las fracturas craneales lineales generalmente vida, que no mejora tras la administración de oxígeno. En la gasometría
no precisan tratamiento quirúrgico, salvo que se traten de fracturas es llamativa la importante hipoxia. Para el diagnóstico es necesario realizar
deprimidas que produzcan lesiones en el parénquima. una Rx de tórax, donde lo más típico es encontrar un infiltrado reticulono-
dular (en vidrio esmerilado) con broncograma aéreo. La imagen radioló-
Pregunta 5.- R: 4 gica no es patognomónica (también aparece en neumonías).
Dentro de los fenómenos dermatológicos que podemos encontrar en un Respuesta 3: corresponde a la taquipnea transitoria del RN.
neonato, y que no tienen significación patológica, se hallan: los quistes Respuesta 4: típico de la displasia broncopulmonar.
de milium, la mancha mongólica, los angiomas planos, el eritema tóxico Respuesta 5: corresponde a un neumotórax bilateral.
y la melanosis pustulosa.
El eritema tóxico es más propio de la raza blanca y consiste en vesiculopús- Pregunta 8.- R: 4
tulas sobre base eritematosa que suele respetar palmas y plantas y tiende a El tratamiento de la EMH consiste en:
desaparecer en la primera semana. La melanosis pustulosa es más frecuente • Monitorización estrecha, control de líquidos.
en la raza negra, la erupción vesiculopustulosa sí suele afectar a las palmas • Intentar mantener una aceptable oxigenación (pO2 50-70 mmHg). Si no
y plantas, y tiene tendencia a desaparecer en varias semanas. Recuerda las se consigue con oxígeno indirecto, será necesario intubar al paciente e
principales diferencias entre eritema tóxico y la melanosis pustulosa. iniciar ventilación mecánica.
• Administración de surfactante intratraqueal en las primeras 24 horas de
ERITEMA TÓXICO MELANOSIS PUSTULOSA vida, pudiendo recibir hasta 4 dosis según evolución. Esta medida se ha
Aparición 1-3 días Nacimiento visto que mejora la clínica de la EMH, pero que no disminuye los casos
de DBP.
· Variable · Variable
Localización • Administración de antibióticos: ampicilina más gentamicina, porque
· No palmoplantar · Sí palmoplantar
Frotis Eosinófilos Neutrófilos la clínica y la Rx pueden ser semejantes en el caso de una sepsis con
neumonía.
Cultivo Estéril Estéril

Pregunta 5. Melanosis pustulosa vs. eritema tóxico del RN. La respuesta incorrecta es la 4. Las prostaglandinas en los neonatos
están indicadas en caso de cardiopatías ductus dependientes, en las
Las manchas hipopigmentadas geográficas se asocian a esclerosis que es IMPRESCINDIBLE mantener el ductus abierto.
tuberosa (respuesta 1).
La ictericia en las primeras 24 horas de vida es siempre patológica (res- Pregunta 9.- R: 2
puesta 2) por hemólisis e infecciones (sepsis, TORCH). El caso clínico es el de un niño que ha sufrido un SAM. Lo típico es que se
El nevus flammeus o mancha“en vino de Oporto”se asocia a angiomatosis produzcan en RN postérmino, con un estrés en el momento del parto, que
encefalotrigeminal o síndrome de Sturge-Weber (respuesta 3). estimula el peristaltismo intestinal y se produce la eliminación de meconio
Manchas café con leche se asocian a neurofibromatosis tipo 1 (res- intraútero.
puesta 5). Al nacimiento, el meconio se encuentra en las vías aéreas altas; si no se elimina
a tiempo, éste pasará al pulmón con las primeras respiraciones, formando
Pregunta 6.- R: 2 tapones que ejercen mecanismo valvular favoreciendo el atropamiento
El cuadro clínico se refiere a una taquipnea transitoria del RN, síndrome de aire y produciendo un cuadro de alveolitis con riesgo de atelectasias y
de Avery, pulmón húmedo o SDR tipo II. sobreinfección bacteriana, siendo el germen más frecuente E. coli. En los
La presentación típica es un RN nacido por cesárea o por parto vaginal casos más graves se asocia con una alta mortalidad debido a la hipertensión
rápido, lo que supone que el tórax no ha estado sometido a la presión pulmonar que desarrollan.
positiva que exprime de líquido los alveolos del pulmón. La clínica se El tratamiento consiste en ventilación mecánica, antibióticos profi-
inicia en las primeras horas de vida, que cede y mejora con medidas lácticos y medidas para disminuir la hipertensión pulmonar: alcalinizar,
poco agresivas (oxígeno en incubadora). Suelen alcanzar la resolución tolazolina y oxido nítrico inhalado. Algunos niños se pueden beneficiar
clínica en un plazo máximo de 3 días. de la ECMO (oxigenación por membrana extracorpórea).
Lo que establece el diagnóstico, aparte de la evolución clínica, es la Rx de
tórax, donde se aprecia aumento de marcas vasculares, líquido en las cisuras. Pregunta 10.- R: 1
La EMH es típica de prematuros, especialmente habitual en hijos de Caso clínico de una parálisis braquial superior, también conocida como pa-
madre diabética, por déficit de surfactante y por inmadurez estructural rálisis de Erb-Duchenne. Se debe a la afectación de las raíces C5-C6 del plexo
pulmonar. Rx de tórax: patrón reticulonodular (vidrio esmerilado) con braquial. El recién nacido presenta el brazo en aducción y rotación interna
broncograma aéreo (respuesta 1 falsa). con el reflejo de Moro ausente en el lado de la lesión (respuesta 1 correcta).
El SAM es típico de RN postérmino, con antecedentes de estrés en Sin embargo, el reflejo de prensión palmar suele estar presente al no verse
el momento del parto. Rx tórax: infiltrados parcheados algodonosos afectadas las raíces inferiores. En los casos en que se lesione también la raíz
(respuesta 3 falsa). C4 puede asociarse con una parálisis frénica.

CTO Medicina • C/Francisco Silvela, 106 • 28002 - Madrid • Tfno. (0034) 91 782 43 30/33/34 • E-mail: secretaria@ctomedicina.com • www. ctomedicina.com 2
Comentarios de Test a distancia 1.ª vuelta
Pediatría
PARÁLISIS BRAQUIAL P. ERB-DUCHENNE P. KLUMPKE La matriz germinal es una estructura transitoria, presente en el cerebro
inmaduro en la cabeza del núcleo caudado, cerca de los ventrículos late-
Raíces (C4) - C5 - C6 C7 - C8 - (T1)
rales hasta la semana 34 de gestación y es muy sensible a la hipoxia. Dicha
Brazo en aducción
Clínica Mano caída matriz está constituida por vasos y su lesión produce hemorragia que se
y rotación interna
diagnostica a través de una ecografía transfontanelar (respuesta 3 correcta).
R. Moro No presente o asimétrico Presente
R. prensión palmar Presente No presente Pregunta 14.- R: 4
Asociaciones C4 - Parálisis frénica T1 - S. Horner Estamos ante un RN con un cuadro de enterocolitis necrotizante (NEC).
La presentación típica es en un RNPT que a la segunda semana de vida
Pregunta 10. Parálisis braquial. inicia cuadro de vómitos, distensión abdominal y deposiciones con sangre.
Dentro de los factores de riesgo figuran, aparte de la prematuridad, la
Pregunta 11.- R: 5 administración precoz y con altas concentraciones de leche, situaciones
Estamos ante un caso típico de displasia broncopulmonar: niño de de hipoxia y bajo gasto y todos aquellos cuadros que favorezcan isquemia
un mes de vida con antecedente de prematuridad y enfermedad de intestinal. La prueba diagnóstica indicada sería una Rx de abdomen
membrana hialina que requiere altas concentraciones de oxígeno para donde se apreciaría el dato más característico, la neumatosis intestinal.
mantener adecuadas saturaciones. Dentro de los factores de riesgo para El tratamiento se debe instaurar ante la mínima sospecha, puesto que
este cuadro figuran: la toxicidad del oxígeno en altas concentraciones, de eso va a depender el pronóstico. Recuerda que la lactancia materna
inmadurez pulmonar y barotrauma. es factor protector. Si el niño no tiene signos de perforación intestinal,
El diagnóstico se establece por la historia clínica y la Rx de tórax, donde el tratamiento será dieta absoluta y antibioterapia que cubra anaerobios
es típico encontrar el llamado patrón de esponja (áreas más claras que y gramnegativos. El tratamiento será quirúrgico en caso de perforación
alternan con otras de mayor densidad). intestinal o sepsis refractaria al tratamiento médico.
El tratamiento se basa en oxigenoterapia, restricción de líquidos y diu- Las complicaciones a largo plazo son la estenosis intestinal y el síndrome
réticos, broncodilatadores y corticoides (sólo en casos graves). Se ha de intestino corto.
demostrado que la vitamina A disminuye la incidencia de DBP.
La mayoría de los niños suelen tener un curso favorable, alcanzando Pregunta 15.- R: 3
la normalidad de la función pulmonar hacia los 2 años de vida. Hay un El cuadro clínico se refiere a un íleo meconial. La forma de presentación
pequeño porcentaje que evoluciona hacia un cuadro de hipertensión más habitual en los neonatos es como un cuadro de obstrucción intes-
pulmonar persistente (respuesta 5 falsa). Las dos causas más frecuentes de tinal congénita. El dato que apoyaría este diagnostico sería palpar unos
muerte en estos niños son la ICC derecha y la bronquiolitis necrotizante. cordones duros que siguen el marco cólico, puesto que el lugar donde
con mayor frecuencia se impacta el meconio es la zona de íleon distal y
Pregunta 12.- R: 3 colón proximal. El diagnóstico se hace mediante radiografía de abdomen
Se trata de un caso típico de hernia diafragmática congénita. La posterior con distensión de asas intestinales y patrón granular espumoso en los
o de Bochdaleck es el tipo de hernia diafragmática más frecuente, sobre puntos de concentración meconial.
todo izquierda. Consiste en un orificio grande en la parte posterior izquier- Una vez establecido el diagnóstico es siempre obligatorio descartar una
da del diafragma a través del que vísceras abdominales migran hacia el fibrosis quística (un 15% debutan como íleo meconial).
tórax, con lo que el abdomen aparece excavado (vacío de contenido) y hay El tratamiento, si el niño no tiene signos de perforación intestinal, se
un compromiso respiratorio grave secundario a la hipoplasia y compresión realiza mediante la administración de enemas hiperosmolares. Si con
pulmonar. Es un cuadro severo que aboca a la instauración de hiperten- esta medida no se soluciona el problema o se producen complicaciones,
sión pulmonar que ya está presente en el nacimiento (respuesta 5 falsa). sería necesario recurrir a la cirugía, realizando una resección de la zona
El diagnóstico en la mayoría de los casos se hace por ecografía prenatal. de impactación y anastomosis terminoterminal.
Cuando no es así, se realiza Rx de tórax al nacimiento donde adverti-
remos la presencia de asas intestinales en tórax, y el mediastino estará Pregunta 16.- R: 3
desplazado por el efecto masa de éstas. Si tras la radiografía hay dudas Respecto a las ictericias neonatales, es importante tener claro varios
con malformaciones pulmonares quísticas, hay que realizar una ecografía aspectos.
(respuesta 3 correcta). • A continuación se reflejan las causas más frecuentes de ictericia
El cierre quirúrgico se hará a las 24-72 horas tras estabilizar la situación según el momento de aparición:
hemodinámica del paciente, primero durante 10-12 horas forzaremos 1.as 24 horas Hemólisis.
una situación de alcalosis (el pH alcalino vasodilata las arterias pulmo- Infecciones: sepsis, TORCH.
nares) para minimizar la hipertensión pulmonar (respuesta 2 falsa). 2.º-3.er día Fisiológica.
Prácticamente todos tienen reflujo gastroesofágico al nacer, y hasta un Infecciones: sepsis, TORCH.
20% necesita una funduplicatura (respuesta 4 falsa). Anemias hemolíticas.
La hernia diafragmática congénita es más frecuente en algunos síndromes 4.º-7.º día Sepsis.
congénitos, como las trisomías 21, 13 y 18. En un 20-30% de los casos TORCH.
hay otras alteraciones asociadas (respuesta 1 falsa) como malrotación Obstrucción intestinal.
intestinal (la más frecuente), cardiopatías y malformaciones vasculares. Lactancia materna.
>1 mes Galactosemia, hipotiroidismo, lactancia materna,
Pregunta 13.- R: 3 metabolopatías, ictericia obstructiva, Gilbert,
Estamos ante caso típico de hemorragia de la matriz germinal: RNPT Crigler-Najjar.
con distrés respiratorio y que de forma brusca desarrolla hipotensión, • Características de la ictericia fisiológica vs. no fisiológica:
bradicardia, cianosis, fontanela abombada y disminución del hematocrito, - Inicio en las primeras 24 horas: SIEMPRE patológico.
el diagnostico será de hemorragia de la matriz germinal. - Predominio de bilirrubina directa: SIEMPRE patológico.

CTO Medicina • C/Francisco Silvela, 106 • 28002 - Madrid • Tfno. (0034) 91 782 43 30/33/34 • E-mail: secretaria@ctomedicina.com • www. ctomedicina. 3
Comentarios de Test a distancia 1.ª vuelta
Pediatría
• Otros datos de la ictericia no fisiológica: Las cifras más bajas se producen a los 2 meses en los RNPT y a los 3 en
- Incremento mayor de 5 mg/24 horas. los RNT (respuesta 1 falsa).
- Bilirrubina en sangre de cordón mayor de 3 mg/dl. El tratamiento consiste en suplementos de hierro a partir de los 2 meses
- RNT > 12 y RNPT > 14. en los RNPT, ya que antes los depósitos están llenos por la hemólisis
- Duración superior a 14 días. de los glóbulos rojos (respuesta 3 correcta), y en casos seleccionados
sería necesario la transfusión de concentrados de hematíes (respuesta
Pregunta 17.- R: 3 4 correcta).
La ictericia por lactancia materna (LM) es la causa más frecuente de ictericia En los neonatos no se manifiesta la betatalasemia, porque la cadena
tardía. Su frecuencia es de aproximadamente 1/200 (respuesta 1 correcta). beta de la hemoglobina se empieza a sintetizar a partir de los 6 meses
Comienza a manifestarse entre el 5.º y el 7.º día de vida alcanzando su (respuesta 5 correcta).
máximo en la tercera semana (respuesta 2 correcta).
Se debe a que en la leche materna existen unas sustancias (pregnanodiol, Pregunta 21.- R: 3
ácidos grasos de cadena larga) que inhiben la glucuronil-transferasa, Es muy importante recordar las situaciones en las que se debe adminis-
dando como resultado un aumento de la bilirrubina a expensas de trar gammaglobulina específica anti-D para prevenir la isoinmunización
fracción indirecta (respuesta 4 correcta). Rh. Es necesario inyectar la gammaglobulina anti-D a las 28 semanas
En ocasiones, cuando existen dudas diagnósticas, se puede interrumpir la de gestación y en las primeras 72 horas después del parto, aborto o
LM transitoriamente durante unos días, observándose un claro descenso amniocentesis, si se confirma que el recién nacido es Rh positivo y la
en la bilirrubina, pero ésta no es indicación de suspender definitivamente madre Rh negativo. La profilaxis se hará sólo si la gestante no ha sido
la LM, puesto que suele ser moderada (valores inferiores a 15) y no se previamente sensibilizada, es decir, sólo si el test de Coombs indirecto
han descrito casos de kernicterus (respuesta 3 falsa). de la gestante es negativo (respuesta 3 falsa).

Pregunta 18.- R: 5 Pregunta 22.- R: 5


Para el MIR es importante saberse bien las diferencias entre la incom- La hipoglucemia neonatal es bastante frecuente en los hijos de madre
patibilidad Rh y AB0. diabética (75% de los hijos de madres diabéticas y 25% de los hijos de
madres con diabetes gestacional). Se debe a que el HMD sintetiza ex-
Rh madre (-), hijo (+) AB0 madre (0) ceso de insulina para compensar el déficit materno de esta hormona y
el riesgo es proporcional a los niveles de glucosa en sangre de cordón.
Frecuencia Menor Mayor Recuerda que puede aparecer en las primeras 48 horas de vida, riesgo
Aparición Después del primer embarazo Primer embarazo máximo entre las 3 y las 6 horas de vida y normalmente cursa de forma
asintomática.
· Más grave (ictericia,
· Leve (ictericia, anemia) El control durante el parto de la glucemia materna junto con el control
Gravedad anemia)
· No hidrops
· Hidrops de la glucemia venosa o capilar en el neonato y el inicio precoz de la ali-
Coombs directo +/-; mentación oral (respuesta 5 falsa), disminuyen el riesgo de hipoglucemia
Diagnóstico Coombs directo +; indirecto +
indirecto + neonatal. El tratamiento consiste en el aporte de glucosa i.v. mediante
· Exanguinotransfusión/ perfusión continua, evitando los bolos por el riesgo de hiperglucemia
fototerapia Fototerapia y/o de rebote.
Tratamiento
· Prevención exanguinotransfusión
con gammaglobulina anti-D
Pregunta 23.- R: 1
Pregunta 18. Diagnóstico diferencial de la isoinmunización El embarazo del hijo de madre diabética es considerado un embarazo de
del recién nacido. alto riesgo, debido al gran número de problemas que puede presentar
el RN. Entre ellos figuran:
Pregunta 19.- R: 4 • Mayor mortalidad fetal y neonatal.
El caso clínico hace referencia a una enfermedad hemorrágica del RN. La • Polihidramnios.
causa es un déficit de vitamina K que produce a su vez déficit de factores • Macrosomía con visceromegalia si la madre no tiene vasculopatía,
k dependientes (respuesta 1 correcta), debido a que durante el emba- si la tiene, CIR.
razo el paso transplacentario de vitamina K es escaso. La LM es pobre • Estenosis subaórtica con hipertrofia septal asimétrica.
en vitamina K (respuesta 4 falsa) y en el intestino no hay bacterias que • Riesgo de EMH: la insulina en altas concentraciones inhibe la síntesis
sinteticen esta vitamina. Todas estas circunstancias obligan a administrar de surfactante.
vitamina K (1 mg i.m.) a todos los RN en las primeras 24 horas de vida. • Policitemia y sus consecuencias (ictericia y trombosis de la vena renal).
La clínica consiste en sangrado a distintos niveles en las primeras 24 • Mayor incidencia de malformaciones congénitas:
horas (respuesta 2 correcta): umbilical, digestivo, nasal, etc. - Malformaciones más frecuentes: cardíacas.
El tratamiento consiste en la administración de nuevas dosis de vitamina - Malformación digestiva más frecuente: colon izquierdo hipoplá-
K o plasma fresco congelado (respuesta 5 correcta). sico.
Es importante recordar que los hijos de madres que durante el embarazo - Malformación más característica: agenesia lumbosacra.
han tomado fenitoína o fenobarbital tienen más riesgo de padecer este • Alteraciones metabólicas:
trastorno (respuesta 3 correcta). - Hipoglucemia: máxima en las 3-6 primeras horas.
- Hipocalcemia.
Pregunta 20.- R: 1
La llamada anemia fisiológica se debe a un déficit transitorio de EPO, Pregunta 24.- R: 3
que unido a la hemólisis de glóbulos rojos durante los primeros meses Recuerda las principales características de la sepsis PRECOZ, TARDÍA y
de vida, dejan al niño anémico (respuesta 2 correcta). NOSOCOMIAL.

CTO Medicina • C/Francisco Silvela, 106 • 28002 - Madrid • Tfno. (0034) 91 782 43 30/33/34 • E-mail: secretaria@ctomedicina.com • www. ctomedicina.com 4
Comentarios de Test a distancia 1.ª vuelta
Pediatría
ETIOLOGÍA CLÍNICA CURSO PRONÓSTICO TRATAMIENTO

· Más afectación respiratoria


· Muy malo
(quejido, polipnea, tiraje,
· S. agalactiae (mueren 30%) Ampicilina + gentamicina
Sepsis precoz aleteo, cianosis), ictericia
· E. coli Fulminante · La listeriosis es muy (ampicilina + cefotaxima,
(3.º a 5.º día) · Listeria: además,
· Listeria monocytogenes grave (mueren si meningitis)
granulomas en faringe,
40-80%)
petequias

· Ampicilina + gentamicina
· Más afectación del SNC
Sepsis tardía · S. agalactiae serotipo III Menos Menos malo · Si meningitis:
(meningitis neonatal)
(7-28 días) · E. coli serotipo K1 fulminante Alta morbilidad - No ingresados: ampicilina
· Focalizan más
+ cefa 3.º

· Vancomicina + amikacina
· S. epidermidis
+ anfotericina B
Sepsis · S. aureus
Variable Variable Variable · Si meningitis:
nosocomial · P. aeruginosa
vancomicina + ceftacidina
· C. albicans
+ anfotericina B

Pregunta 24. Sepsis neonatal.

Pregunta 25.- R: 5 y calcificaciones periventriculares (en el caso de la toxoplasmosis, éstas


La madre que puede contagiar una hepatitis B al RN es aquella que son difusas).
tiene el Ags +, (hepatitis crónica activa, hepatitis aguda o portadora). Si La infección congénita por CMV es la infección viral congénita más fre-
además el Age es positivo, el riesgo asciende hasta un 90%. El momento cuente (respuesta 1 falsa). El 90% de los recién nacidos están asintomáticos
de mayor riesgo es el momento del parto. (respuesta 2 correcta), pero si tienen clínica pueden presentar ictericia,
La mayoría de las veces, la enfermedad en el neonato es asintomática, hepatoesplenomegalia, calcificaciones, hepatitis,etc. La neumonitis es
pero tiene una alta probabilidad de evolucionar hacia una forma crónica la forma más característica de infección postnatal.
y sufrir degeneración maligna. La secuela más frecuente es la hipoacusia neurosensorial, no de trans-
Para evitar este curso es fundamental realizar profilaxis con gammaglo-bulina misión (respuesta 3 falsa).
específica en las 12 primeras horas de vida y primera dosis de vacuna. Si la La afectación ocular típica es la coriorretinitis (respuesta 4 falsa).
profilaxis se realiza correctamente, podrá recibir lactancia materna. Poste- Estos niños suelen desarrollar microcefalia evolutivamente (respuesta
riormente hay que seguir la pauta de vacunación de VHB a los 2 y 6 meses. 5 falsa).

Pregunta 26.- R: 2 Pregunta 27.- R: 2


El caso clínico nos presenta a un RN con clínica de CMV congénita. Los Recuerda que si la infección de la gestante por toxoplasma ocurre en el
datos claves que nos permiten llegar al diagnóstico son: coriorretinitis primer trimestre, el RN puede presentar la tétrada de Sabin consistente

IgGCTOMEGALOVIRUS TORMOPLAMA
• Se reactiva asintomática 1-2%
Infección materna • Se infecta
• Se infecta sintomática 1-2%0
• Placentaria 1.er trimestre más frecuente, más grave • Placentario
Transmisión • Canal de parto 1.er trimestre más grave
• Lactancia 3.er trimestre más frecuente
• 5-18% secuelas tardías, hipoacusia neurosensorial bilateral
Ansiolítico más frecuente Sin tratamiento coriorretinitis
y severa.
SABIN
• Corriorretinitis • Coriorretinitis
Síntoma • Microcefalia • Hidrocefalia
• Calcificaciones periventriculares • Calcificaciones intracraneales difusas
• Convulsiones
1.ª determinación:
• Negativo: vigilar
Screening prenatal • Prenatal Ac antiCMV (no se hace prevención) • Positivo: hacer una 2.ª determinación:
- Disminuido igual: infección pasada
- Aumantado: tratar
• Aislamiento y cultivo en orina “inclusiones en ojo de búho” • Aislamiento en placenta (¿sangre?)
Diagnóstico • IgM • IgM
• IgG o estable a las 6 semanas • IgG o estable a las 6 semanas
Siempre:
No • Sintomático: primetadina + sulfadicina 6 meses
Tratamiento
• Ganciclovir a veces primetadina + sulfamicina/espiromicina 6 meses
• Asintomático: espiromicina y valorar serología

Pregunta 27. Diferencias entre toxoplasma y citomegalovirus.

CTO Medicina • C/Francisco Silvela, 106 • 28002 - Madrid • Tfno. (0034) 91 782 43 30/33/34 • E-mail: secretaria@ctomedicina.com • www. ctomedicina. 5
Comentarios de Test a distancia 1.ª vuelta
Pediatría
en coriorretinitis), manifestación más habitual, aunque suele aparecer • Al contrario que en todas las infecciones connatales, la rubéola tiene
de forma tardía calcificaciones intracraneales difusas, hidrocefalia y más riesgo de trasmisión en el 1.er trimestre. También es más grave
convulsiones. si se infecta en este trimestre.
Si ocurre en el segundo o tercer trimestre, la infección fetal es más • La tríada de Gregg se caracteriza por:
frecuente, pero el RN suele presentar síntomas leves o enfermedad - Hipoacusia neurosensorial: es la manifestación más habitual.
subclínica. - Cardiopatía: la más frecuente, el DAP.
- Afectación ocular: lo más habitual son las cataratas.
Pregunta 28.- R: 1 • Un RN infectado por rubéola congénita puede eliminar el virus en
Estamos ante un niño RN CIR (edad gestacional de 38 semanas y peso las secreciones hasta año y medio después del nacimiento, por lo
< 2.500 g) con clínica de erupción petequial y hepatoesplenomegalia. que es obligatorio aislarlo de mujeres embarazadas.
Esto lo puede presentar cualquier infección connatal.
A continuación nos describen la clínica típica del DAP: soplo continuo Pregunta 29.- R: 1
en 2.º espacio intercostal izquierdo con pulsos femorales saltones. Es un De los factores que se citan, el más relacionado con el CIR tipo simétrico
caso de rubéola congénita. Las lesiones óseas también son las típicas (CIR tipo I) es la infección congénita.
de la rubéola, las que nos encontraríamos en la sífilis serían las mismas, Recuerda que hay que sospechar una infección connatal ante la presen-
pero con reacción perióstica. cia de CIR tipo simétrico, hepatoesplenomegalia, adenopatías, ictericia,
anemia y trombopenia en un recién nacido.
La causa más frecuente de CIR tipo asimétrico (CIR tipo II) es la insufi-
ciencia placentaria.

Pregunta 30.- R: 4
Todo el caso clínico hace referencia a la infección por el virus herpes
simple, siendo el más frecuente el tipo II (75-90%).
La clínica suele comenzar a la semana, con grave afectación del estado
general, fontanela abombada y vesículas en la zona de presentación (en
la práctica clínica sólo aparecen en un 70% de los casos). Se asocia con
una elevada mortalidad. El neonato contrae la infección cuando pasa por
el canal del parto, por lo tanto, lesiones genitales activas en el momento
del parto contraindican un parto vaginal y es indicación de cesárea.
El tratamiento en este niño sería la administración de aciclovir intravenoso.
No te olvides de repasar las características de las infecciones connatales.

Pregunta 31.- R: 2
El granuloma, que aparece al caerse el cordón (por infección leve o por
epitelización incompleta), es un tejido blando, granular, vascular y rojizo
Pregunta 28. Manifestaciones clínicas de la rubéola congénita. o rosado, que puede tener a veces una secreción mucopurulenta.
De la rubéola congénita para el MIR debemos recordar: Las masas umbilicales se pueden clasificar en grandes (hernia umbilical,

MOMENTO VÍA
ESTIGMAS CARACTERÍSTICOS
DE ADQUISICIÓN DE ADQUISICIÓN
· Tríada de Gregg: catarata, sordera/cardiopatía (ductus sobre todo)
Rubéola Sobre todo 1.er trimestre Placentaria · Otros: coriorretinitis, retinopatía sal-pimienta, estenosis pulmonar
periférica, púrpura trombopénica
Más frecuente en 3.er trimestre · Calcificaciones periventriculares cerebrales, microcefalia, coriorrenitis
CMV Placentaria, canal,
Más grave en 1.er trimestre: más · Muchos asintomáticos (con o sin secuelas tardías: la más frecuente es
(la más frecuente) leche
clínica la sordera)
· Precoz: CIR, vesículas, alteraciones neurológicas y oculares
Canal,
VHS (queratoconjuntivitis)
Más frecuente en parto transplacentario,
(75-95% VHS II) · Tardía (en parto): sepsis, encefalitis con lesión del lóbulo temporal,
postparto (90%)
alteración ocular (sobre todo queratoconjuntivitis y vesículas cutáneas)
· Calcificaciones cerebrales periféricas
Más grave en 1.er trimestre, más
Toxoplasmosis Placentaria · Tétrada de Sabin: coriorretinitis, hidrocefalia, convulsiones,
frecuente en el 3.er trimestre
calcificaciones
Más grave en 1.er trimestre. Si se · Precoz: cicatrices, atrofia de miembros, malformaciones oculares
Varicela presenta en el 3.er trimestre, es más Placentaria y cerebrales
grave cuanto más cerca del parto · Tardía: vesículas, afectación visceral y dificultad respiratoria
· Precoz (< 2 años): pénfigo sifilítico, hepatoesplenomegalia, rinitis (tríada
de la sífilis precoz)
Sífilis Sobre todo en 3.er trimestre Placentaria
· Tardía (> 2 años): tríada de Hutchinson (sordera + queratitis
+ alteraciones dentarias), articulación de Clutton, periostitis

Pregunta 30. Infecciones connatales.

CTO Medicina • C/Francisco Silvela, 106 • 28002 - Madrid • Tfno. (0034) 91 782 43 30/33/34 • E-mail: secretaria@ctomedicina.com • www. ctomedicina.com 6
Comentarios de Test a distancia 1.ª vuelta
Pediatría
onfalocele y gastrosquisis) y pequeñas (granuloma, pólipo y onfalitis). Su densidad es mayor que la leche humana madura (respuesta 3 correcta)
Recuerda las principales características de cada uno de ellos. pues contiene mayor cantidad de proteínas y minerales que ésta pero no
hidratos de carbono (respuesta 2 falsa) ni grasa, y una serie de factores
Hernia umbilical Onfalocele Gastrosquisis inmunitarios importantes en la defensa del RN (respuesta 5 correcta).
DEFECTO DE Paramedio
Con los días, el calostro es sustituido por una leche de transición que
CIERRE
Umbilical Umbilical se convierte en madura hacia la 3.ª-4.ª semana (respuesta 4 correcta).
(lat. dcho. + frec.)
CUBIERTAS Peritoneo y piel Peritoneo Sin peritoneo
Pregunta 36.- R: 2
- Anomalías
- Estrangulación cromosómicas Es importante recordar las principales características que las dife-
(rara) - Extrofia vesical rencian.
COMPLICACIONES Atresia intestinal
- Reducción - Sd. Beckwith
espontánea (macrosomía e
hipoglucemia) LECHE HUMANA LECHE DE VACA

Quirúrgico, si: Calorías 670 kcal/l 670 kcal/l


- Estrangulación Corrección
Corrección · 1-1,5 g · 3-4,5 g
- Crecimiento quirúrgica precoz
TRATAMIENTO quirúrgica precoz Proteínas · Caseína 30% · Caseína 80%
progresivo (buen
(peor pronóstico) · Seroproteínas 70% · Seroproteínas 20%
- Persiste a los pronóstico)
3-5 años Hidratos de carbono 7 g lactosa y otras 4,5 g lactosa

Pregunta 31. Patología umbilical. · 3,5 g % ác. grasos · 3,5 g


esenciales · Escasos ác. grasos
Grasas · Ác. grasos cadena larga esenciales
Sobre secreciones por el ombligo han preguntando en el MIR: insaturados · Ác. grasos
• Persistencia del uraco: se segrega por el ombligo un líquido amari- · Colesterol saturados
llento similar a la orina con un pH ácido. Minerales + +++ (5 veces)
• Persistencia del conducto onfalomesentérico (fístula intestinal):
Hierro + +
aparece un líquido amarillento con un pH alcalino.
Cobre ++ +

Pregunta 32.- R: 4 Flúor + -


El hipotiroidismo congénito es una patología relativamente común que Relación calcio/fósforo 2 1
se debe descartar en todos los neonatos. Su causa más frecuente es la Vitamina A ++ +
disgenesia tiroidea (respuesta 1 correcta).
Vitamina B + ++
En el recién nacido los signos y síntomas de hipotiroidismo son muy
sutiles y la clínica se va instaurando de forma progresiva (respuesta Vitamina C + Escasa
4 falsa). Aparece facies peculiar, estreñimiento, ictericia prolongada, Vitamina D + Escasa
letargia, hernia umbilical, fontanelas amplias, retraso en la madura- Vitamina E ++ +
ción ósea y puede asociarse una sordera neurosensorial (respuesta Vitamina K Escasa +
2 correcta).
Nitrógeno
La determinación de TSH en sangre, obtenida entre los 2 y los 5 días de no proteico
+++ +
vida, es la prueba de cribado rutinaria. Cuando se confirma el diagnóstico
es necesario iniciar el tratamiento precoz con levotiroxina para evitar la
aparición de retraso mental irreversible (respuesta 3 correcta). Pregunta 36. Comparación entre leche humana
y leche de vaca.
Pregunta 33.- R: 4
Recordar los siguientes datos de forma muy simplificada: Pregunta 37.- R: 3
• 1,5 m: sonrisa social. La causa más frecuente de talla baja en el niño son las dos variantes de
• 3 m: inicio sostén cefálico. la normalidad.
• 6 m: inicia sedestación.
• 9 m: movimiento de pinza. TALLA RETRASO CONSTITUCIONAL
• 10-12 m. Inicia bipedestación. BAJA FAMILIAR DEL CRECIMIENTO
• 12-15 m: primeros pasos y emite bisílabos.
Menor de la
• 24 m: sube/baja escaleras, corre y apila 4 o 6 cubos para formar una Talla RN Normal
normal
torre.
Velocidad
• 5-10 años: comprende que la muerte es un fenómeno permanente. de crecimiento
Normal Menor de lo normal, luego normal

Pregunta 34.- R: 5 Antecedentes


De talla baja De pubertad retrasada
familiares
Recuerda que el peso del niño se duplica al 5.º mes, se triplica al año y
se cuadriplica a los 2 años. La talla del nacimiento se duplica alrededor · Igual a edad talla
Igual a edad
Edad ósea · Retrasada respecto a la edad
de los 4 años de edad (respuesta 5 falsa). cronológica
cronológica

Pregunta 35.- R: 2 Talla final Disminuida Normal o algo disminuida


Del calostro debes saber que es la leche de los 2-4 primeros días después
del parto (respuesta 1 correcta). Pregunta 37. Tipos de talla baja no patológica.

CTO Medicina • C/Francisco Silvela, 106 • 28002 - Madrid • Tfno. (0034) 91 782 43 30/33/34 • E-mail: secretaria@ctomedicina.com • www. ctomedicina. 7
Comentarios de Test a distancia 1.ª vuelta
Pediatría
Pregunta 38.- R: 5 seguridad (UCI, quirófano) y administración de antibióticos, general-
Si tenemos presente la tabla del comentario de la pregunta anterior mente cefalosporinas de 3.ª generación. Los corticoides pueden ser
podemos deducir fácilmente que estamos ante un caso de talla baja beneficiosos en la primera parte del tratamiento pero la adrenalina no
familiar. La curva de crecimiento es inferior al percentil 3, no hay dis- es útil (respuesta 3 falsa). Con el tratamiento adecuado, el cuadro suele
cordancia entre la edad ósea del niño y su edad cronológica y, además, remitir en 24-48 horas.
existe historia familiar de talla baja. Las pruebas de laboratorio serán En los casos dudosos nunca hay que intentar visualizar la epiglotis,
normales y la talla adulta final previsible será baja, pero dentro de los sino realizar una Rx lateral de faringe donde se apreciará una epiglotis
límites esperados para su talla genética. engrosada.

Pregunta 39.- R: 3 Pregunta 42.- R: 5


Es un cuadro de laringitis aguda, obstrucción de la vía aérea superior, de Estamos ante un cuadro de bronquiolitis aguda que se define como
etiología viral (virus parainfluenzae en el 75% de los casos) caracterizado primer episodio de dificultad respiratoria con sibilancias. El agente más
por un antecedente previo de infección de vías altas con fiebre de 38-38,5 frecuente es el VRS. Es una enfermedad estacional de los meses de in-
ºC, tos perruna o metálica, afonía, estridor de predominio inspiratorio y vierno y primavera.
dificultad respiratoria variable con empeoramiento nocturno. Afecta a niños menores de 2 años, principalmente lactantes. Comienza por
Afecta a niños entre 3 meses y 5 años y tiene predominio estacional, un cuadro catarral, con tos, mocos y febrícula, y en los días siguientes evolu-
produciéndose la mayoría de las veces en otoño. ciona hacia un cuadro de obstrucción de vías aéreas distales con sibilancias
El tratamiento consiste en humedad ambiental, corticoides (en aerosol (dato más típico de la exploración), espiración alargada, roncus dispersos e
o sistémicos) y adrenalina en aerosol. hipoventilación. Esta fase crítica suele durar 2-3 días con posterior mejoría
(respuestas 1 y 3 correctas). En lactante muy pequeño pueden producirse pau-
Pregunta 40.- R: 2 sas de apnea con cianosis como única manifestación (respuesta 2 correcta).
Es un cuadro clínico típico de laringitis pero sin fiebre y sin ningún an- El tratamiento consiste en medidas generales (oxigenoterapia, monito-
tecedente de infección de vías altas de características virales. Se trata, rización, etc.), beta-2 agonistas inhalados y adrenalina nebulizada. Ni los
por tanto, de una laringitis espasmódica o estridulosa. corticoides ni los antibióticos están indicados.
Es el momento de repasar las características de los tipos de crup. La ribavirina inhalada estaría recomendada en bronquiolitis graves
que afectan a niños de riesgo (patología pulmonar o cardíaca de base).
Pregunta 41.- R: 3 Actualmente se intenta prevenir su aparición con la administración de
La epiglotitis es una enfermedad típica de los niños menores de 5 años. palivizumab, anticuerpo monoclonal anti-VRS indicado en prematuros,
En la actualidad, los principales responsables de esta entidad son los cardiópatas, neumópatas e inmunodeprimidos.
cocos grampositivos como el S. pyogenes, S. pneumoniae y S. aureus. Por La secuela a largo plazo más frecuente es la hiperreactividad bronquial.
detrás de éstos, hay que tener en cuenta también a su agente clásico, La bronconeumonía y OMA son poco frecuentes (respuesta 5 falsa).
H. influenzae tipo B (cada vez menos frecuente gracias a la vacunación
universal). Pregunta 43.- R: 4
La clínica suele cursar de forma brusca por la noche, con fiebre alta y El cuadro que presenta este niño es compatible con una fibrosis quística.
aspecto séptico, babeo e intensa dificultad respiratoria con estridor Las manifestaciones clínicas suelen ser:
inspiratorio. Empeora con el llanto y con el decúbito supino, haciendo • Respiratorias: bronquiolitis recurrentes, tos crónica, neumonía, pólipos
que el niño permanezca en “posición de trípode”. No suele existir tos ni nasales, sinusitis, etc.
afonía y el estridor no es tan ruidoso como el de las laringitis. • Digestivas: íleo meconial, esteatorrea, malabsorción, prolapso rectal,
En el tratamiento, lo prioritario es asegurarse la vía aérea mediante cirrosis biliar, etc.
intubación (a ser posible nasotraqueal), siempre en condiciones de • Otras: deshidratación, malnutrición, diabetes, azoospermia.

LARINGITIS AGUDA TRAQUEÍTIS


LARINGOTRAQUEITIS VIRAL EPIGLOTITIS AGUDA
(estridulosa) BACTERIANA

Virus parainfluenzae 1
Etiología Alergia + psicológico Cocos grampositivos S. aureus
(el más frecuente)

Catarro vías altas


Antecedentes No hay - Crup vírico
(paciente y/o familiar)

Espasmo laríngeo recortado


Fiebre, tos, disnea alta, estridor Fiebre alta + babeo + disfagia +
Clínica (generalmente nocturno) Fiebre + estridor mixto
inspiratorio disnea + cabeza extendida
No fiebre

Duración 1-2 noches Días-semanas Fulminante Días-semanas

· Oxígeno
· Ambiente tranquilo, humidificar · Intubar + oxígeno
· Intubación si es
Tratamiento · Corticoides · Antibiótico
preciso
· Adrenalina racémica en aerosol · Tto. en UVI
· Cloxacilina i.v.

Pregunta 40. Tipos de crup.

CTO Medicina • C/Francisco Silvela, 106 • 28002 - Madrid • Tfno. (0034) 91 782 43 30/33/34 • E-mail: secretaria@ctomedicina.com • www. ctomedicina.com 8
Comentarios de Test a distancia 1.ª vuelta
Pediatría
Para el diagnóstico se requiere manifestaciones clínicas compatibles y/o Pregunta 48.- R: 1
antecedente de hermano/a con enfermedad confirmada y/o cribado Estamos ante un caso de alergia a proteínas de leche de vaca. El cuadro
neonatal positivo más un criterio de confirmación (test del sudor positivo típico consiste en inicio de la clínica tras la ingesta de un biberón de
o presencia de mutaciones en el estudio genético o test de diferencia fórmula adaptada de leche de vaca. La proteína más frecuentemente
de potencial nasal positivo). implicada es la betalactoglobulina (respuesta 2 correcta). La sintoma-
Para el tratamiento de estos pacientes es fundamental el cuidado del tología más habitual es la cutánea (urticaria), seguida de la digestiva
pulmón. Se debe realizar diariamente fisioterapia respiratoria y trata- (vómitos, diarrea) y en ocasiones respiratoria (bronco espasmo).
miento agresivo de las infecciones pulmonares. Además, se llevará a El cuadro está mediado por un mecanismo inmunológico tipo IgE.
cabo un adecuado soporte nutricional y tratamiento específico de las Afecta a individuos atópicos o con antecedentes familiares de atopia
posibles complicaciones. (respuesta 1 falsa).
El diagnóstico es fundamentalmente clínico, se basa en la exclusión
Pregunta 44.- R: 1 de dichas proteínas y consecuente desaparición de los síntomas y la
La traqueítis bacteriana tiene como principal responsable a S. aureus posterior provocación (respuesta 4 correcta). Se emplean las pruebas
(respuesta 1 falsa). Hay que sospechar esta entidad cuando, tras un cutáneas, que serán positivas (respuesta 3 correcta).
cuadro de crup vírico, se produce un empeoramiento progresivo con El tratamiento consiste en la suspensión de la fórmula adaptada, reem-
fiebre, dificultad respiratoria de intensidad creciente y estridor mixto. plazándola por fórmulas especiales (hidrolizado de proteínas). Suele ser
Es una entidad grave que suele requerir la hospitalización del niño para un proceso autolimitado (respuesta 5 correcta), la mayoría de los niños
instaurar tratamiento antibiótico (cloxacilina i.v.) y oxigenoterapia a pueden tomar leche de vaca a partir de los 2 años.
demanda. Si aparece gran dificultad respiratoria puede ser necesaria Recuerda las diferencias entre INTOLERANCIA y ALERGIA a las proteínas
la intubación. de leche de vaca.

Pregunta 45.- R: 4 APLV IPLV


En la patogenia de la fibrosis quística está implicada una alteración de
Patogenia Mediado IgE No IgE
la regulación de los canales iónicos de las membranas celulares, lo que
produce secreciones deshidratadas y espesas. A nivel pulmonar, la le- Síntomas Digestivos y alérgicos Digestivos
sión anatomopatológica inicial es la bronquiolitis (respuesta 1 correcta). Clínico
El íleo meconial supone la forma de debut neonatal en aproximadamente Provocación
Diagnóstico
un 10% de los casos (respuesta 2 correcta). La función endocrina del · InmunoCAP
páncreas se afecta con el tiempo, pudiendo aparecer diabetes mellitus a · Prick
partir de los 10 años de evolución de la enfermedad (respuesta 3 correcta). HIDROLIZADOS DE PLV
Debido a la pérdida excesiva de sal, se ven cuadros de deshidratación Tratamiento · AntiH1
con hiponatremia e hipocloremia, coincidiendo con gastroenteritis o en · Adrenalina s.c.
épocas de calor (respuesta 5 correcta).
La aspergilosis broncopulmonar alérgica es relativamente frecuente en Pregunta 48. Intolerancia-alergia a proteínas de leche de vaca.
estos pacientes. Hay que sospecharla ante la presencia de esputo herrum-
broso, aislamiento de Aspergillus fumigatus o la presencia de eosinófilos Pregunta 49.- R: 4
en una muestra fresca de esputo. El tratamiento será corticoterapia oral, Caso clínico de atresia con fístula traqueoesofágica. Según la clasifi-
en los casos refractarios puede ser necesario el empleo de anfotericina cación de Ladd, el tipo más frecuente es la atresia proximal con fístula
B en aerosol o de 5-fluorocitosina sistémica (respuesta 4 falsa). distal (tipo III).
El diagnóstico se realiza por la sospecha clínica (salivación excesiva,
Pregunta 46.- R: 4 cianosis y atragantamiento con las tomas, etc.) e imposibilidad para
Es necesario iniciar el tratamiento de la fibrosis quística antes de que pasar una sonda nasogástrica.
la afectación pulmonar sea importante. Se ha visto que el empleo de El tratamiento es quirúrgico, siendo el reflujo gastroesofágico la com-
fisioterapia respiratoria alarga la supervivencia de estos pacientes. plicación postquirúrgica más frecuente, que suele ser grave (respuesta
Es útil el empleo moderado de enzimas pancreáticas para disminuir la 4 correcta). Otras complicaciones menos habituales son la fístula de
esteatorrea, así como el aporte de suplementos de vitaminas liposolu- la anastomosis, la recidiva de la fístula traqueoesofágica, la estenosis
bles a la dieta. esofágica y la traqueomalacia.
En los pacientes con fibrosis quística, la neumonía por sobreinfección
bacteriana se debe principalmente a Pseudomonas aeruginosa, variedad Pregunta 50.- R: 5
mucoide. Este patógeno puede colonizar la vía respiratoria dando lugar Estamos ante un caso de RGE fisiológico, caracterizado por pequeños
a un estado de portador crónico difícil de revertir (respuesta 4 falsa). vómitos sin fuerza después de las tomas, por lo demás, el niño se en-
cuentra asintomático con peso y talla en percentiles normales.
Pregunta 47.- R: 4 El RGE es muy frecuente en el primer año de vida, hasta un 85%. Gene-
La causa más frecuente de GEA en nuestro medio es el rotavirus. Es una ralmente desaparece hacia los 2 años.
enfermedad típica de lactantes que produce una diarrea líquida, SIN En un 10% de los casos, se habla de RGE patológico o enfermedad por
sangre. Es proceso autolimitado, que cede en 3-10 días (respuesta 4 reflujo gastroesofágico (ERGE) cuando se asocia con escasa ganancia
falsa). El diagnóstico se basa en la realización de un test de ELISA en las ponderal, esofagitis y clínica respiratoria.
heces para detectar el Ag. El diagnóstico se basa en la historia clínica. Sólo en el RGE patológico está
El tratamiento es sintomático. Existen dos tipos vacunas atenuadas indicada la realización de pruebas complementarias: tránsito superior
de 2 o 3 dosis respectivamente de reciente aparición y administra- con bario (primera prueba a realizar) y una pHmetría de 24 horas (para
ción oral. cuantificar el grado de reflujo es la prueba más sensible y específica, se

CTO Medicina • C/Francisco Silvela, 106 • 28002 - Madrid • Tfno. (0034) 91 782 43 30/33/34 • E-mail: secretaria@ctomedicina.com • www. ctomedicina. 9
Comentarios de Test a distancia 1.ª vuelta
Pediatría
ESTENOSIS HIPERTRÓFICA
REFLUJO GASTROESOFÁGICO ATRESIA PILÓRICA ATRESIA DUODENAL
DE PÍLORO
Momento de aparición 1.ª semana 2.ª a 3.ª semana
Vómitos Alimentarios sin fuerza Alimentarios a chorro Alimentarios Biliosos
• Síntomas respiratorios • Masa palpable • Abdomen excavado
Otros síntomas
• Disminución del crecimiento • Alcalosis hipoclorémica • Asociar al Down
Clínica + respuesta a tratamiento
 Evolución RX: RX:
Diagnóstico pHmetría 24 h • Rx: “signo de la cuerda” • 1 burbuja gástrica • 1 burbuja gástrica
 • 1 burbuja gástrica • Ausencia de gas distal • 1 burbuja duodenal
endoscopia
• Medidas posturales Cirugía:
Tratamiento • Cisaprida • Piloromiotomía Cirugía Cirugía
• Cirugía (funduplicatura de Nissen) • Fredet-Ramsted

Pregunta 51. Vómitos del lactante.

realiza cuando existen manifestaciones extradigestivas del reflujo). El lactantes (respuesta 1 correcta). Se localiza a unos 50-70 cm de la válvula
tratamiento en los casos leves consiste en medidas posturales y espe- ileocecal (respuesta 2 correcta).
santes de las tomas. En casos de reflujos más graves se administraran La clínica más frecuente suele consistir en una hemorragia rectal IN-
estimulantes del peristaltismo (cisaprida, domperidona) o tratamiento DOLORA (respuesta 4 falsa) e intermitente debido a la ulceración de la
quirúrgico (funduplicatura de Nissen). mucosa ileal adyacente al divertículo con mucosa ectópica (gástrica o
pancreática).
Pregunta 51.- R: 5 La técnica diagnóstica más sensible es la gammagrafía con Tc-99.
La clínica típica de un cuadro de estenosis hipertrófica de píloro consiste El tratamiento es quirúrgico.
en vómitos proyectivos inmediatos tras las tomas, alimenticios, no biliosos,
tras el vómito el niño se queda irritable y hambriento. Comienza entre Pregunta 54.- R: 3
las 3-6 semanas de vida, siendo el momento más habitual en torno a los Cuadro típico de invaginación intestinal. Recuerda que es la causa más
20 días de vida (respuesta 1 falsa). Es más habitual en varones de raza frecuente de obstrucción intestinal entre los 3 meses y los 6 años. Niño
blanca (respuesta 2 falsa). que presenta episodios repetidos de llanto y encogimiento de piernas
Debido a las pérdidas de hidrogeniones y cloruros con los vómitos se con posterior decaimiento. Si la clínica evoluciona hasta un 60%, puede
produce una alcalosis metabólica hipoclorémica (respuesta 3 falsa) con presentar deposiciones“en jalea de grosella”, heces con sangre roja fresca
normopotasemia o hipopotasemia. y moco. La forma más frecuente es la ileocólica.
La prueba diagnóstica de elección ante la sospecha de este cuadro sería Para el diagnóstico inicial se suele realizar una ecografía abdominal.
una ecografía abdominal (respuesta 4 falsa). El hallazgo frecuente en la Una invaginación intestinal es siempre una urgencia. Si han transcurrido
radiografía simple de abdomen es la distensión gástrica con escaso gas menos de 48 horas y no hay signos de perforación, se realiza tratamien-
distal. El tránsito digestivo está en desuso en la actualidad, se reserva to con enemas de bario o con aire, para intentar la reducción. Si hay
para pacientes en los que la ecografía no es concluyente, donde aparece perforación intestinal o distensión abdominal de más de 48 horas de
de forma típica la imagen del “signo de la cuerda”. evolución, se prefiere tratamiento quirúrgico.
El tratamiento es quirúrgico: pilorotomía extramucosa de Ramsted (res- La tasa de recurrencia es mayor en los pacientes en los que se realiza
puesta 5 correcta). Recuerda que pueden existir vómitos postoperatorios reducción hidrostática (10-20%) que en los tratados quirúrgicamente
secundarios al edema de píloro producidos por la propia incisión. (3%).
Repasa los cuadros clínicos que producen vómitos en el lactante.
Pregunta 55.- R: 4
Pregunta 52.- R: 4 La enfermedad de Hirschprung o megacolon congénito se caracteriza
Caso típico de atresia duodenal con vómitos biliosos en las primeras 24- por estreñimiento crónico desde el nacimiento, asociado a un estanca-
48 horas, tras las primeras tomas, con abdomen excavado por ausencia miento o retraso ponderal (por malabsorción) secundario a un trastorno
de aire distal. Importante el antecedente de síndrome de Down, más en la migración de los neuroblastos que supone la ausencia de células
común en estos niños. Con mayor frecuencia la atresia se localiza a nivel ganglionares y plexos mesentéricos.
de la 3.ª porción del duodeno. En la Rx de abdomen es típico encontrar Para el diagnóstico es útil el tacto rectal, donde se aprecia una am-
la imagen de “doble burbuja”. Recuerda los datos clínicos característicos: polla rectal vacía de heces (respuesta 3 correcta). En la manometría
• Estenosis hipertrófica de píloro: vómitos no biliosos a las 2-3 semanas se observa un aumento paradójico del tono del esfínter anal interno
de vida. (respuesta 4 falsa) ante un incremento de presión a ese nivel. En el
• Enfermedad de Hirschprung: estreñimiento y en ocasiones vómitos enema opaco se objetiva un retraso en la eliminación de contraste
fecaloideos. (respuesta 5 correcta). El diagnóstico definitivo lo dará la biopsia: en el
• Divertículo de Meckel: sangrado indoloro rectal sin vómitos. segmento afectado se observa ausencia de células ganglionares con
• Invaginación intestinal: episodios de dolor intenso, siendo muy raro incremento de la acetilcolinesterasa y aumento de las terminaciones
en el periodo neonatal. nerviosas (respuesta 1 correcta).
El tratamiento es quirúrgico, con la resección de todo el segmento
Pregunta 53.- R: 4 agangliónico. En el RN con obstrucción intestinal se realiza colostomía
El divertículo de Meckel es un resto del conducto onfalomesentérico. y descenso con posterior cierre de colostomía. Si no hay obstrucción,
Es la malformación digestiva más frecuente, apareciendo en un 2% de se realiza un descenso primario precoz.

CTO Medicina • C/Francisco Silvela, 106 • 28002 - Madrid • Tfno. (0034) 91 782 43 30/33/34 • E-mail: secretaria@ctomedicina.com • www. ctomedicina.com 10
Comentarios de Test a distancia 1.ª vuelta
Pediatría
Pregunta 56.- R: 5 Se hará, al menos en una ocasión, estando el paciente consumiendo
Los factores que intervienen en la patogenia de la celiaquía son: gluten (respuesta 4 falsa). Una vez asegurado el diagnóstico se iniciará
• HLA: DR3, DR4, DR7 , DQ y W2. el tratamiento a base de una dieta rigurosamente exenta de gluten de
• Factores inmunológicos: los linfocitos de la lámina propia se inmu- por vida.
nizan frente a la gliadina. La biopsia intestinal en el caso de la enfermedad celíaca no es patog-
• Factores ambientales. nomónica (respuesta 3 falsa), pero sí muy característica: aparece un
infiltrado inflamatorio en la lámina propia, junto con hiperplasia de las
La clínica se caracteriza por un cuadro de malabsorción, de inicio más criptas y atrofia vellositaria.
frecuente entre los 6 meses y los 2 años. Suelen presentar esteatorrea, El seguimiento se realizará valorando la evolución clínica y determinando
estancamiento de la curva ponderal, disminución de la masa muscular, periódicamente los anticuerpos para evaluar el cumplimiento de la dieta
irritabilidad y distensión abdominal importante. (respuesta 5 falsa). Los anticuerpos más sensibles y específicos son los
El diagnóstico se basa en los test serológicos, Acs antigliadina, antirre- IgA antitransglutaminasa (respuesta 2 correcta).
ticulina, antiendomisio y antitransglutaminasa, estos últimos los más
sensibles y específicos de todos, y en la realización de biopsia intestinal, Pregunta 58.- R: 2
necesaria para establecer diagnóstico. La biopsia se hará, al menos, en Estamos ante un cuadro de atresia de vías biliares extrahepáticas (AVBEH);
una ocasión, estando el paciente consumiendo gluten y se tomarán lo típico es RN a término sano y con fenotipo normal que desarrolla
varias muestras de porción duodenal o inicio de yeyuno. Clásicamente cuadro de colestasis (predominio de bilirrubina directa, coluria, acolia
se realizaban tres biopsias: 1.ª (sin exclusión), demuestra atrofia; 2.ª (tras o hipocolia) a las 2-3 semanas de vida. Se puede asociar a poliesplenia,
exclusión), demuestra recuperación de mucosa; y 3.ª (tras prueba de malrotación intestinal, anomalías vasculares, etc. La aparición de varios
provocación), demuestra atrofia. Actualmente la 2.ª y la 3.ª se realizan casos en la misma familia es rara.
únicamente cuando: La prueba diagnóstica sería la realización de un HIDA, donde se aprecia
• El inicio de la clínica se produjo por debajo de los 2 años de edad captación del contraste por el hígado, pero que no llega al intestino (au-
(riesgo de falsos positivos en la biopsia). sencia de excreción). El diagnóstico de certeza se consigue mediante la-
• La respuesta clínica a la exclusión del gluten de la dieta no ha sido parotomía exploradora con realización intraoperatoria de colangiografía.
concluyente y existe duda diagnóstica. Su tratamiento definitivo es el trasplante hepático (el 80% de pacientes
• La retirada del gluten se hizo sin realización de primera biopsia. lo requerirá a lo largo de su evolución). Previamente se intentará realizar
una hepatoportoenterostomía o técnica de Kasai, para intentar restable-
Para llevar a cabo la 2.ª biopsia, se debe esperar un mínimo de 24 meses cer el flujo biliar y disminuir el daño (respuesta 2 falsa). Actualmente, la
tras retirada del gluten y no ha de practicarse hasta los 6 años de vida. AVBEH constituye la primera indicación del trasplante hepático infantil.
El tratamiento consiste en la exclusión de por vida del trigo, cebada, cen-
teno y +/- avena y triticale. Pueden tomar libremente maíz, soja y arroz.

Pregunta 58. Colestasis neonatal.

Pregunta 59.- R: 3
Es un cuadro de malabsorción de hidratos de carbono. Tiene como an-
Pregunta 56. Diagnóstico de la enfermedad celíaca. tecedente una GEA que lesiona el borde del enterocito, produciendo un
déficit transitorio de lactasa. La clínica se caracteriza por diarrea, dolor
Pregunta 57.- R: 2 cólico, heces espumosas y ácidas (pH < 5,5) que excorian la zona del
Pregunta importante sobre el diagnóstico de la enfermedad celíaca. Ante pañal. Ante la sospecha se debe realizar un Clinitest en las heces que
la presencia de una alta sospecha clínica es necesaria la realización de será positivo. El tratamiento consiste en la exclusión de la lactosa de la
una biopsia intestinal para confirmar el diagnóstico (respuesta 1 falsa). dieta, que la mayoría de las veces es transitoria.

CTO Medicina • C/Francisco Silvela, 106 • 28002 - Madrid • Tfno. (0034) 91 782 43 30/33/34 • E-mail: secretaria@ctomedicina.com • www. ctomedicina. 11
Comentarios de Test a distancia 1.ª vuelta
Pediatría
• El déficit de sacarasa-isomaltasa: produce la misma clínica pero el Pregunta 63.- R: 4
Clinitest es negativo. Caso clínico típico de síndrome hemolítico urémico (SHU); niño
• El déficit aislado de isomaltasa: es asintomático. menor de 4 años con antecedente de gastroenteritis enteroinvasiva
• El déficit de enteroquinasa: produce una malabsorción de proteínas que presenta signos y síntomas de insuficiencia renal aguda, clínica
con clínica de desnutrición y edemas. neurológica y afectación gastrointestinal. El agente más frecuente es E.
• Gastroenteritis por rotavirus: la clínica es vómitos y diarrea líquida, coli O-156). El patógeno produce unas toxinas que favorecen la lesión
no ácida. a nivel endotelial capilar y arteriolar con la formación de trombos de
plaquetas con trombopenia de consumo y anemia microangiopática
Pregunta 60.- R: 2 (respuesta 2 correcta). El órgano más afectado en este cuadro es el
La criptorquidia (descenso testicular incompleto) es el trastorno de la riñón y es la causa más frecuente de IRA en menores de 5 años (res-
diferenciación sexual más frecuente en los varones. Se estima que un puesta 1 correcta).
4,5 % de los niños tienen criptorquidia al nacimiento. Como el descenso El diagnóstico se basa en:
testicular se produce en la última fase de la gestación, es más frecuente • Anemia hemolítica microangiopática: haptoglobina descendida con
en prematuros. La mayoría de los testículos no descendidos bajan al esquistocitos en el frotis de sangre periférica.
escroto espontáneamente en los primeros 3 meses de vida. Si ésto no • Plaquetopenia.
ha ocurrido a los 6 meses de edad, es poco probable que lo haga más • IRA: por ecografía se debe diferenciar de la trombosis venosa bilateral
adelante. Las consecuencias de la criptorquidia permanente son: (respuesta 3 correcta).
• Riesgo de esterilidad en los casos bilaterales.
• Riesgo de degeneración maligna a seminoma. Se realizará tratamiento conservador de la IRA y alteraciones hidro-
• Hernias asociadas. electrolíticas y si no consiguen controlarse, estará indicada la diálisis
• Torsión testicular en el lado afecto. peritoneal (respuesta 4 falsa).
• Efectos psicológicos. La función renal se recupera en el 90% de pacientes y las recidivas son
poco frecuentes.
La edad idónea para realizar el tratamiento es entre los 9 y los 15 meses
de edad, preferentemente antes de los 2 años. Consiste en el descenso y Pregunta 64.- R: 2
la fijación quirúrgica del testículo al escroto (orquidopexia); en todo caso Estamos ante un cuadro de torsión testicular. Se debe realizar diagnóstico
es primordial realizarla antes de la pubertad, sobre todo para eliminar el diferencial con la orquioepididimitis.
riesgo de degeneración maligna. El tratamiento con HCG es más eficaz La clínica consiste en un dolor agudo intenso, con tumefacción escrotal, sin
cuanto mayor es la edad del niño y con una localización baja del teste. fiebre ni traumatismo previo. El reflejo cremastérico suele estar anulado.
El diagnóstico se basa fundamentalmente en la clínica, en caso de duda
Pregunta 61.- R: 4 se puede recurrir a la gammagrafía de flujo testicular o a la realización
En un lactante que presenta un cuadro clínico inespecífico (vómitos, de una eco-Doppler.
febrícula y pérdida de apetito), junto con leucocitosis, leucocituria y El tratamiento consiste en la reducción manual o fijación quirúrgica del
nitritos positivos en orina, debemos sospechar una infección del tracto teste afecto y del contralateral.
urinario. Recuerda que la tira reactiva de orina es un método sensible El tiempo de viabilidad del testículo torsionado es de 4-6 horas, por lo
que permite seleccionar aquellos niños a los que hay que realizar un que siempre se debe considerar una urgencia quirúrgica.
cultivo de orina, en este caso obtenido mediante punción suprapúbica, Recuerda las causas de escroto agudo.
e iniciar el tratamiento antibiótico.
Reflejo
Dolor Estado general Inflamación
Pregunta 62.- R: 3 cremastérico
Respecto al RVU, hay que saber que es la anomalía congénita más fre- · Intenso · Afectación · Abolido · Teste indurado
cuente de la unión ureterovesical. Se suele diagnosticar al estudiar las TORSIÓN
· Brusco importante · Teste y edematoso
ITU porque es causa predisponente de la misma. El paso retrógrado de · Continuo · Cortejo ascendido · Eritema
TESTICULAR
vegetativo o escrotal
orina desde la vejiga al uréter y pelvis renal desencadena una reacción transverso
inflamatoria que puede dar lugar a la formación de cicatrices renales y
· Menos Mínima Presente · Menos
si éstas son extensas, se afectará la función renal (nefropatía por reflujo, intenso afectación intensa
que es la causa de hasta un 20% de las insuficiencias renales y la primera TORSIÓN · Gradual · Punto azul
causa de HTA en la infancia). DEL · Dolor a la en polo
Para el diagnóstico, la técnica de elección es la cistografía miccional que HIDÁTIDE palpación superior
del polo
permite clasificar el reflujo según intensidad, grado de dilatación ureteral superior
y deformidad calicial (grados I-V). Se debe realizar también una ecografía
· Menos Síndrome Presente · Moderada
renal para descartar anomalías estructurales y una gammagrafía renal, ORQUIOEPI-
intenso miccional · Edema
DIDIMITIS
técnica de referencia para el diagnóstico de cicatrices renales. · Progresivo asociado escrotal
El reflujo grado I y II en el 80% de los casos desaparece de forma espon-
tánea al madurar el niño y no precisan tratamiento. Los objetivos del
tratamiento son prevenir la pielonefritis y la lesión renal. Pregunta 64. Diagnóstico diferencial del escroto agudo.
• La profilaxis antibiótica a dosis básica es el pilar fundamental del
reflujo leve hasta que éste desaparezca. Pregunta 65.- R: 3
• En los casos que por su grado y repercusiones morfológicas a nivel Caso clínico de PTI (púrpura trombótica idiopática). Típicamente apa-
renal, se espera que no desaparezca y/o lesione más el riñón, se rece como un cuadro de petequias generalizadas, con disminución
obtará por la cirugía, que puede ser endoscópica o abierta. del número de plaquetas, tras un catarro de vías altas. No suele existir

CTO Medicina • C/Francisco Silvela, 106 • 28002 - Madrid • Tfno. (0034) 91 782 43 30/33/34 • E-mail: secretaria@ctomedicina.com • www. ctomedicina.com 12
Comentarios de Test a distancia 1.ª vuelta
Pediatría
anemia ni otros signos de diátesis hemorrágica. El pronóstico es bueno, el año y los 5 años. Se han encontrado deleciones en el cromosoma 11
evolucionando hacia la recuperación espontánea en la mayoría de los (respuesta 1 correcta).
casos (respuesta 3 cierta). La clínica más frecuente es la masa abdominal asintomática (respuesta 5
correcta) que no suele pasar línea media. Otros síntomas son la HTA, en
Pregunta 66.- R: 4 el 60% de pacientes por compresión arterial por el tumor y producción
Es importante que recuerdes: de renina (respuesta 3 correcta), hematuria, etc.
• Cáncer infantil más frecuente: leucemias (LAL tipo B). Los linfomas La localización más común de las metástasis es el pulmón.
siguen a las leucemias en frecuencia, salvando el grupo de edad de El diagnóstico se basa en la TC y la PAAF, no aconsejándose la realización
entre 10 y 14 años, en que superan a las anteriores. de biopsia (respuesta 4 falsa).
• Cáncer sólido infantil más frecuente: tumores del SNC (astrocitoma).
• Tumor sólido extracraneal más frecuente: neuroblastoma. El neuro- Pregunta 69.- R: 5
blastoma, es más habitual en varones y en la raza blanca, y el 90% Ante un niño que debuta de forma brusca con un síndrome constitu-
se diagnostican por debajo de los 5 años de edad. cional junto con pancitopenia y hepatoesplenomegalia, el diagnóstico
• Tumor abdominal más frecuente: neuroblastoma. más probable será el de leucemia aguda (respuesta 5).
• Masa abdominal más habitual en el RN: hidronefrosis.
• En cuanto a los tumores óseos, en niños mayores y adolescentes, Pregunta 70.- R: 4
predomina el osteosarcoma, seguido del sarcoma Ewing, que es el El tumor que tiene tendencia a presentar calcificaciones intraneoplásicas
más frecuente en menores de10 años. (hasta en un 80% de los casos) es el neuroblastoma y no el tumor de
Wilms (respuesta 4 falsa).
Pregunta 67.- R: 3 Con las pruebas de imagen se puede confirmar que el tumor de Wilms
El cuadro que se expone es una niña con un neuroblastoma. La edad de (nefroblastoma) es de origen intrarrenal (respuesta 2 correcta). En oca-
aparición típica es en menores de 2 años. Se suele localizar en el abdo- siones, los pacientes con nefroblastoma pueden presentar policitemia
men a nivel de las glándulas suprarrenales y la clínica más frecuente es la debido a la producción de eritropoyetina (respuesta 1 correcta).
masa abdominal que cruza línea media. Se asocia a hematoma palpebral Las metástasis pulmonares son las más frecuentes dentro del tumor
y hepatomegalia y entre los síndromes paraneoplásicos (no modifican de Wilms, pero su presencia no suele contraindicar la cirugía del tumor
el pronóstico) figura el opsoclonus-mioclonus (caso clínico) y la diarrea (respuesta 3 correcta).
secretora. Para el diagnóstico se utiliza la TC craneal, catecolaminas en El tumor de Wilms puede ser bilateral, sobre todo en las formas familiares
orina aumentadas y la gammagrafía con MIBG. (5%), (respuesta 5 correcta).
El tratamiento depende del estadio, realizándose cirugía, quimioterapia
y radioterapia. La supervivencia es del 50%. Recuerda las principales Pregunta 71.- R: 4
diferencias entre neuroblastoma y nefroblastoma. Estamos ante un cuadro de sarampión. El caso clínico va relatando
detalladamente las fases de esta enfermedad:
NEUROBLASTOMA WILMS (NEFROBLASTOMA) • Fase prodrómica: catarro con tos y fiebre, manchas de Koplik (lesiones
blanquecinas sobre base eritematosa en mucosa subyugal), es un
· Tumor sólido · 2.º tumor abdominal más
extracraneal más frecuente en niños dato patognomónico de esta enfermedad.
Epidemiología frecuente en niños · Mayores que los niños del • Fase exantemática: inicio del exantema por la cara y descenso (res-
· Sobre todo < 2 años neuroblastoma (≥ 3 años) puesta 1 correcta). El exantema es maculopapular, no pruriginoso
· Deleción cromosoma 1 · Deleción cromosoma 11 (respuesta 4 falsa) y la gravedad de la enfermedad se relaciona con
· 70% abdomen la intensidad y confluencia del exantema (respuesta 3 correcta). Es
(suprarrenal · Riñón en esta fase cuando aparece fiebre alta.
Localización o paramedial) · No pasa línea media
· Pasa línea media · A veces bilateral (familiares) • Fase de resolución: el exantema desaparece en el mismo orden que
· 20% tórax apareció.
· Asocia a veces
hemihipertrofia, aniridia Entre las complicaciones figuran:
· Masa (sd. de Beckwith- • Cuadros ORL (otitis media aguda): complicación más frecuente.
· Sd. paraneoplásicos Wiedemann) y
Clínica • Neumonías: en los niños es más común la sobreinfección bacteriana,
(VIP, opsoclono- malformaciones
mioclono) genitourinarias que la neumonía de células gigantes de Hecht (respuesta 2 correcta).
· Masa abdominal • Afectación del SNC: encefalitis aguda y PEES (panencefalitis es-
· HTA clerosante subaguda): forma de encefalitis por virus lentos, con
· Bueno. A veces se diferencia un pronóstico pésimo, el diagnóstico se hace detectando en LCR
· Cirugía en estadio I o regresa espontáneamente aumento de Acs frente al sarampión.
Tratamiento
· Otros: Qx + QT + RT · Metástasis a hígado, m.o.,
• Anergia cutánea con reactivación de una tuberculosis preexistente
piel y hueso
(respuesta 5 correcta).
· Qx · Bueno, sobre todo en
Pronóstico · RT menores de 2 años
· QT (según estadio) · Metástasis a pulmón Pregunta 72.- R: 3
Con los datos de febrícula y adenopatías retroauriculares y cervicales
Pregunta 67. Neuroblastoma vs. nefroblastoma. dolorosas debemos pensar en una rubéola. Se precede de un cuadro
catarral leve (1 o 2 días) con fiebre baja o moderada y conjuntivitis. El
Pregunta 68.- R: 4 exantema es morbiliforme y confluente en cara. Se resuelve mediante
El tumor de Wilms se asocia con anomalías genitourinarias, hemihipertro- una mínima descamación. En la analítica se puede encontrar leucopenia,
fia y aniridia (respuesta 2 correcta). La edad de diagnóstico oscila entre trombopenia y linfocitos atípicos.

CTO Medicina • C/Francisco Silvela, 106 • 28002 - Madrid • Tfno. (0034) 91 782 43 30/33/34 • E-mail: secretaria@ctomedicina.com • www. ctomedicina. 13
Comentarios de Test a distancia 1.ª vuelta
Pediatría
El período de máxima transmisión abarca desde 7 días antes de apari- • Tercera fase, con aclaramiento central (aspecto en encaje o reticu-
ción del exantema hasta 7 u 8 días después de que éste haya aparecido. lado). Se resuelve espontáneamente entre la 1.ª y 4.ª semana, pero
Las complicaciones son poco frecuentes en la infancia: artritis (preferente- en ocasiones puede recidivar cuando el niño se estresa, tiene fiebre,
mente articulaciones de pequeño tamaño), encefalitis y púrpura trombo- con el sol, etc.
pénica. La rubéola suele darse en niños mayores de 6 meses, pues antes
los anticuerpos maternos actúan como protectores (respuesta 3 correcta). Entre las complicaciones figuran las artralgias-artritis y la aplasia medular
Una vez superada la enfermedad suele quedar inmunidad permanente. en niños susceptibles, con enfermedades hematológicas, (respuesta 5
correcta).
Pregunta 73.- R: 1
El cuadro clínico descrito es el típico de mononucleosis infecciosa. La Pregunta 76.- R: 2
clínica característica es astenia importante, fiebre alta, adenopatías, Caso clínico de exantema súbito (roséola infantil o sexta enfermedad). Su
faringoamigdalitis con exudado blanquecino en sábana, hepatoespleno- agente es el herpes virus tipo 6 y afecta principalmente a niños menores
megalia y puede cursar con exantema maculopapuloso de predominio de 2 años. Se caracteriza por:
en tronco, a veces desencadenado por la toma de amoxicilina. En la • Fase febril: fiebre alta, con buen estado general, que dura de 3 a 4
analítica típicamente aparece leucocitosis con linfocitosis, presencia de días.
linfocitos atípicos, leve trombopenia y discreto aumento de las transa- • Fase exantemática, aparece bruscamente, tras desaparecer la fiebre,
minasas, pero puede cursar con pancitopenia como es el caso. El virus un exantema maculopapuloso poco confluente que afecta al tórax,
Epstein-Barr, los virus A y B de la hepatitis, el parvovirus B19, el VIH y el abdomen y raíz de miembros.
citomegalovirus, son los más asociados a pancitopenia. Para confirmar el
diagnóstico habría que pedir serología específica, aunque el tratamiento Son datos típicos del hemograma la presencia de leucocitosis con
es sintomático, y en cuanto a la pancitopenia, actitud expectante, pues neutrofilia en las primeras 24-36 horas de evolución, por lo que, en un
se resuelve espontáneamente en la mayoría de las ocasiones. primer momento, puede ser difícil establecer el diagnóstico de infección
La pancitopenia puede ser congénita o adquirida. La más frecuente es vírica (respuesta 2 correcta).
esta última, y las causas más importantes son los fármacos, los tóxicos, La complicación más frecuente es la crisis febril. También puede causar
las infecciones, radiaciones y procesos inmunitarios. Las leucemias y la encefalitis.
hemoglobinuria paroxística nocturna (HPN) son etiologías más raras.
Pregunta 77.- R: 4
Pregunta 74.- R: 1 Recordemos los criterios diagnósticos de la enfermedad de Kawasaki o
El caso que se nos presenta es una varicela. Ya sabemos que el agente sd. mucocutáneo-ganglionar.
causal es el virus de la varicela-zóster, de la familia de los herpes virus.
La fase prodrómica consiste en un catarro de vías altas con febrícula. Lo A Fiebre
característico del exantema es que suele afectar a tronco y parte proximal
de extremidades, respetando las zonas distales. Las lesiones aparecen en Presencia de, como mínimo, cuatro de los cinco siguientes signos:
distintos estadios evolutivos (o lesiones en “cielo estrellado”); éstas son · Conjuntivitis bilateral no purulenta
· Alteraciones en la mucosa de la orofaringe, con inyección faríngea;
lesiones eritematosas con vesículas y otras en fase de costra. Las lesio-
labios secos con fisuras, inyectados o ambos, y lengua “en fresa”
nes pican mucho. La complicación más frecuente es la sobreinfección
B · Alteraciones en las zonas periféricas de las extremidades, como
bacteriana de las lesiones.
edema y eritema en manos o pies, descamación de inicio
periungueal
Otras complicaciones son: · Exantema, de inicio en el tronco; polimorfo, no vesicular
• Neumonía varicelosa: en la infancia es más frecuente la secundaria · Linfadenopatía cervical unilateral
a sobreinfección bacteriana, que la producida por el propio virus.
C La enfermedad no se puede explicar por ninguna otra causa conocida
• Afectación del SNC: lo más habitual es en forma de cerebelitis, que
evoluciona de forma favorable. Diagnóstico clínico A + B + C

La varicela en los niños sanos no precisa tratamiento con aciclovir. Son Pregunta 77. Criterios diagnósticos de la enfermedad de Kawasaki.
indicaciones para su uso: neonatos e inmunodeprimidos.
Además de estos síntomas, se puede encontrar patología muy variada:
Pregunta 75.- R: 3 artritis, pericarditis, iritis, etc. Es muy llamativa la trombocitosis, espe-
La pregunta hace referencia al llamado eritema infeccioso, megaloeritema cialmente en la fase subaguda de la enfermedad.
o quinta enfermedad. En el MIR lo suelen preguntar para confundirlo Recuerda que la etiología es desconocida aunque se plantea que pue-
con el exantema súbito, roséola infantil o sexta enfermedad. de ser secundaria a un daño inmunitario del endotelio mediado por
El eritema infeccioso tiene como agente causal el parvovirus B19 (res- superantígenos.
puesta 1 correcta). Es típico de la edad escolar (5-15 años) y predomina El tratamiento consiste en el empleo de AAS y gammaglobulina en la fase
en meses de primavera y verano. El periodo de incubación es 1-2 se- aguda, manteniendo posteriormente el AAS en dosis antiagregantes. El
manas (respuesta 2 correcta). El cuadro se caracteriza por un periodo empleo de la gammaglobulina en la fase aguda previene el desarrollo
prodrómico que puede cursar con febrícula, sin fiebre alta (respuesta 3 de aneurismas coronarios en la fase subaguda.
falsa), seguido de la clínica exantemática afebril que evoluciona en tres
etapas (respuesta 4 correcta): Pregunta 78.- R: 2
• Fase inicial con eritema en ambas mejillas (¨niño abofeteado¨). Este paciente presenta una púrpura de Schölein-Henoch. Es la vasculitis
• Fase intermedia de exantema maculopapuloso en tronco, sin afec- más frecuente de la infancia. Afecta a pequeños vasos y está mediada
tación de palmas y plantas. por mecanismo inmunológico de predominio IgA.

CTO Medicina • C/Francisco Silvela, 106 • 28002 - Madrid • Tfno. (0034) 91 782 43 30/33/34 • E-mail: secretaria@ctomedicina.com • www. ctomedicina.com 14
Comentarios de Test a distancia 1.ª vuelta
Pediatría
La fase prodrómica suele consistir en proceso ORL. Posteriormente puede Pregunta 81.- R: 5
aparecer, en orden de frecuencia: Respecto al SIDA en pediatría, hay que recordar:
• Afectación cutánea: púrpura palpable, en miembros inferiores, que • La principal vía de transmisión es vertical, durante el parto. El por-
suele cursar en brotes. centaje de infectados es aproximadamente del 1%, si se aplican
• Artritis de rodillas y tobillos. medidas de actuación oportunas. Recuerda que existe transmisión
• Dolor abdominal cólico, vómitos y deposiciones con sangre. documentada del virus mediante leche materna.
• Afectación renal: hematuria con o sin proteinuria. Éste es el factor • El periodo de latencia es menor que en adulto. Se subdivide en SIDA
que marca el pronóstico. precoz (período de latencia de pocos meses, clínica antes del año de
• Sólo en menos de un 1% se produce afectación del SNC. vida, predominando las manifestaciones neurológicas, pronóstico
malo) y SIDA tardío (más parecido al del adulto, predominando la
El tratamiento consiste en antiinflamatorios para las molestias clínica infecciosa).
articulares y en ciclos cortos de corticoides, en caso de dolor ab- • Las manifestaciones clínicas más frecuentes en pediatría, compa-
dominal intenso. Si existe afectación renal intensa, se emplean rándolo con los adultos son: cuadros ORL, parotiditis, neumonía
inmunosupresores. intersticial linfoide y manifestaciones neurológicas (calcificaciones
de los ganglios basales, atrofia cerebral, etc.). La causa más frecuente
Pregunta 79.- R: 2 de muerte es la neumonía por P. carinii.
Para el diagnóstico de mononucleosis infecciosa, aparte de la sospecha • Son infecciones menos frecuentes en los niños: TBC, hepatitis B,
clínica, podemos encontrar: linfomas y sarcoma de Kaposi.
• Leucocitosis (10.000-20.000) con más de un 20-40% de linfocitos
atípicos (respuesta 2 falsa). Pregunta 82.- R: 1
• Aumento de las transaminasas (80% de casos) (respuesta 1 correcta). El diagnóstico en el hijo de madre VIH positiva es difícil por la presencia de
• Anticuerpos heterófilos: son útiles en mayores de 4 años; por debajo IgG anti-VIH de la madre que pasan a través de la placenta, de forma que
de esta edad, la sensibilidad es inferiror al 20%. Puede permanecer el 100% de los niños son seropositivos al nacer. Los niños no infectados
positivo hasta 9 meses después, por lo que no es útil para el diag- ¨falsamente positivos¨ pueden tardar en negativizar anticuerpos 18 me-
nóstico de infección activa. ses (respuesta 1 falsa), por ello, el diagnóstico va a depender de la edad:
• Anticuerpos específicos contra el VEB (respuesta 3 correcta): en fase Recuerda que es necesario realizar quimioprofilaxis con AZT durante el
aguda: la IgM ACV (frente a la cápside viral) se incrementa rápido y embarazo, el momento del parto y luego al RN. El tratamiento se basa en
se mantiene 4 semanas. La IgG ACV aparece al final de la fase aguda la triple terapia, al igual que en los adultos, y para evaluar la evolución,
y se mantiene casi toda la vida. se emplea la carga viral y el cociente CD4/CD8, recordando que la cifra
de linfocitos hay que extrapolarla según la edad del niño.
EL VEB se ha asociado con distintos tumores como el linfoma Burkitt,
carcinoma nasofaríngeo, síndrome de Duncan y diversos síndromes MENORES DE 18 MESES MAYORES DE 18 MESES
linfoproliferativos (respuesta 4 correcta). En la fase aguda de la infec-
ción puede producirse rotura esplénica ante un traumatismo, por ello, 1. PCR-ADN + al menos en 1. Presencia de Ac frente a VIH
el tratamiento consiste en reposo; en algunos casos puede ser útil la 2 determinaciones (ELISA o Western-Blot)
administración de corticoides (obstrucción de la vía aérea alta, anemia 2. Criterios SIDA 2. Criterios del apartado anterior
hemolítica autoinmune, convulsiones, etc.).
Pregunta 82. Diagnóstico de infección VIH en el niño.
Pregunta 80.- R: 4
Estamos ante un cuadro de tos ferina, infección por Bordetella pertussis. Pregunta 83.- R: 3
La edad con más riesgo es el periodo de lactante, la madre no le da La triple vírica protege contra el virus del sarampión, rubéola y parotiditis.
anticuerpos protectores (porque éstos van disminuyendo a lo largo Es una vacuna de virus vivos atenuados.
de la vida) y el niño todavía no tiene completa la vacunación frente al Indicaciones: a los 15 meses y una dosis de recuerdo a los 4 años, a
microorganismo (respuesta 1 correcta). niños y niñas.
El periodo de incubación oscila entre 5-10 días (respuesta 5 correcta). Efectos adversos: cuadro de fiebre y exantema, más atenuados que las
La clínica consta de dos fases: infecciones frente a las que protege (son virus vivos).
• Fase catarral inicial (respuesta 2 correcta): con febrícula, rinorrea y Contraindicaciones:
tos blanda. Este es el periodo de máxima contagiosidad. • Inmunodeprimidos: excepto en niños VIH+ (sólo si existe inmuno-
• Fase de tos paroxística, consiste en accesos de tos, seguido en mu- depresión importante).
chas ocasiones de vómitos y acompañándose de cianosis facial. En • Embarazo.
los niños mayores es característico el “gallo” inspiratorio. Esta fase • La alergia al huevo ha dejado de ser una contraindicación absoluta
puede durar 2-4 semanas (respuesta 3 correcta). (sólo si existe anafilaxia).

La tos ferina, pese a ser infección bacteriana, cursa con linfocitosis Pregunta 84.- R: 4
absoluta (respuesta 4 falsa) y en la radiografía de tórax los datos son Actualmente existen dos tipos de vacunas: polio oral (Sabin) y polio
inespecíficos (infiltrados perihiliares). El diagnóstico se basa en el cul- intramuscular (Salk).
tivo del moco nasal en medio de Bordet-Gengou pero la detección en La polio Sabin es una vacuna de virus vivos atenuados que produce in-
sangre de IgG antifactor estimulante de los linfocitos es la técnica más munidad local tipo IgA y vacunación comunitaria por contacto fecal-oral.
sensible y específica. Como efecto adverso más importante destacan algunos casos aislados
Para el tratamiento se utilizan macrólidos, actualmente, de elección la de polio en niños sanos, lo que ha motivado en los últimos años que se
azitromicina. estén potenciando las vacunas de virus inactivados.

CTO Medicina • C/Francisco Silvela, 106 • 28002 - Madrid • Tfno. (0034) 91 782 43 30/33/34 • E-mail: secretaria@ctomedicina.com • www. ctomedicina. 15
Comentarios de Test a distancia 1.ª vuelta
Pediatría
Está contraindicada en personas inmunodeprimidas y personas que
conviven con éstos.
La vacuna tipo Salk (i.m.) está elaborada con virus inactivados, es
menos eficaz, pero más segura. Es la que actualmente recomienda la
AEP. La pauta de administración es a los 2-4-6-18 meses y a los 4 años.

Pregunta 85.- R: 5
Lo básico que tenéis que recordar de la vacuna DTP es lo siguiente:
• Componentes:
- Difteria: toxoide.
- Tétanos: toxoide.
- Pertussis: dos tipos: celular y acelular. Esta última con muchos
menos efectos adversos a nivel neurológico. Recomendada por
la AEP. La Pertussis celular estaba contraindicada en mayores
de 7 años, porque a partir de esa edad hay más riesgos que
beneficios.
• Indicaciones: en el calendario vacunal se administra a los 2-4-6 y 18
meses y a los 4 años se puede administrar DT o DTPa , luego cada
10 años dT. Las primeras dosis se pueden administrar en un mismo
preparado con otras vacunas (polio inactivada, Hib, VHB), para mi- Pregunta 89. Algoritmo de actuación frente a contacto
nimizar el número de pinchazos. con un bacilífero positivo.

En pacientes inmunodeprimidos se puede administrar sin riesgo. Pregunta 90.- R: 3


El SMSL, muerte repentina de un niño menor de un año cuya etiología se
Pregunta 86.- R: 4 desconoce, supone la causa principal de muerte entre el mes de vida y el
La vacuna del Hib es una vacuna conjugada, muy segura y eficaz. año de edad en los países desarrollados. Este síndrome se correlaciona
Protege frente a infecciones invasivas (meningitis y epiglotitis). En el con la posición de decúbito prono y lateral para dormir. Tras este factor
calendario vacunal se incluye a los 2-4-6 y 18 meses. Estaría indicada posicional, el tabaquismo materno es actualmente el principal factor de
en niños menores de 5 años. Presenta efectos secundarios leves y de riesgo del SMSL. Otros factores son la prematuridad, el antecedente de
corta duración. Contraindicaciones: en edades inferiores a 2 meses por hermano muerto por SMSL, sexo masculino, etc.
no ser inmunógena. Debido a la ausencia de tratamiento, es muy importante la prevención
con medidas como la posición de decúbito supino para dormir y la lac-
Pregunta 87.- R: 4 tancia materna (que se considera el factor preventivo más importante).
La vacuna frente a la hepatitis B está realizada por ingeniería genética.
Producen en las personas que la reciben anticuerpos frente al antígeno Pregunta 91.- R: 5
de superficie del virus. En el calendario vacunal se puede administrar con El ductus arterioso persistente es una cardiopatía típica de RNPT, que
diversas pautas: (0-1-6), (0-2-6) y (2-4-6). Para prevenir la transmisión de consiste en la unión entre la aorta distal a la subclavia izquierda y la
la hepatitis B en el neonato, en el caso de que la madre sea portadora, se arteria pulmonar. Esta cardiopatía congénita también se asocia a la
debe asociar con gammaglobulina específica al nacimiento. rubéola congénita y es más frecuente en niñas que en niños.
Está indicada a cualquier edad si el niño no ha sido vacunado previamente La clínica consiste en un soplo continuo, en maquinaria o de Gibson,
(respuesta 4 falsa). Sus contraindicaciones son muy escasas. Recordar más audible en 2.º espacio intercostal izquierdo; los pulsos femorales
que el embarazo no es una contraindicación absoluta, pero se aconseja son saltones. El niño puede presentar clínica de ICC.
evitar todo tipo de vacunas siempre que sea posible. En la Rx de tórax se aprecia plétora pulmonar con aumento de cavidades
izquierdas. El diagnóstico definitivo lo obtenemos por ecocardiografía.
Pregunta 88.- R: 4 En los RNPT, si no existen contraindicaciones, se comienza con la admi-
Quimioprofilaxis frente a los agentes más frecuentes de meningitis. nistración de indometacina, y si no se soluciona, tratamiento quirúrgico.
• Meningococo: a todos los contactos, independiente de la edad, Recuerda las contraindicaciones de la indometacina: hemorragia activa,
con rifampicina en dosis de 10 mg/k/dosis cada 12 horas, 4 dosis trombopenia, enterocolitis necrotizante e insuficiencia renal. En caso de
+/- vacuna en caso del meningococo C. no ser un RNPT, el tratamiento de entrada es quirúrgico. Cuando interesa
• Hib: niños menores de 5 años sin vacunar y adultos que conviven mantenerlo abierto se utiliza PGE1 i.v. (respuesta 5 falsa).
con menores de 5 años sin vacunar (éstos pueden ser portadores):
rifampicina en dosis de 20 mg/k/dosis cada 24 horas, 4 dosis +/-va- Pregunta 92.- R: 4
cuna. La TGA es la cardiopatía cianógena más frecuente de inicio en el perío-
• Neumococo: no existe quimioprofilaxis activa. Se puede indicar do neonatal (recordad que la más común en conjunto es la tetralogía
la vacuna del neumococo de 7 serotipos para prevenir futuros de Fallot, cuya clínica es más tardía, en torno al año de vida). La aorta
contactos. se origina en el ventrículo derecho, a la derecha y delante de la arteria
pulmonar, que se forma en el ventrículo izquierdo.
Pregunta 89.- R: 2 La clínica suele consistir en un RN que en las primeras horas de vida pre-
En el esquema siguiente aparece esquematizada la actitud que hay que senta taquipnea y cianosis intensa (conforme se va cerrando el ductus),
llevar a cabo con un niño que ha tenido un contacto con un bacilífero no tiene soplo y los pulsos son débiles. En la Rx de tórax se aprecia un
positivo. pedículo cardíaco estrecho con plétora pulmonar. El ECG suele ser de

CTO Medicina • C/Francisco Silvela, 106 • 28002 - Madrid • Tfno. (0034) 91 782 43 30/33/34 • E-mail: secretaria@ctomedicina.com • www. ctomedicina.com 16
Comentarios de Test a distancia 1.ª vuelta
Pediatría
características normales para un RN. El diagnóstico se confirma mediante de crecimiento ni de la edad ósea. La etiopatogenia no está clara, y es
la ecografía. probable que sea multifactorial. Es más frecuente en los dos primeros
El tratamiento consiste en la perfusión de PGE de forma urgente para años de vida, y en general tiene buen pronóstico, con regresión espon-
mantener abierto el ductus, asegurando comunicación entre ambas tánea en la mayoría de los casos en 2-3 años. Sin embargo, a veces es
circulaciones en paralelo para que sea posible la supervivencia y, cuanto persistente, pudiendo evolucionar a una pubertad precoz o adelantada,
antes, realizar la corrección quirúrgica. La técnica de elección es el switch sin que se conozcan actualmente los factores de riesgo para ello. Para
arterial o Jatene, que consiste en seccionar la salida de los grandes vasos el diagnóstico son útiles:
e intercambiarlos. • Rx de mano y muñeca izquierdas (edad ósea).
• Ecografía pélvica.
Pregunta 93.- R: 2 • Niveles basales de estradiol.
Los malos tratos en la infancia son la segunda causa de muerte en • La prueba diagnóstica para excluir una pubertad precoz es el test
España durante los primeros 5 años de vida, excluyendo el periodo de GnRH (respuesta de FSH y LH ante el estímulo con análogos de
neonatal. Es muy importante detectarlos precozmente pues hay casos GnRH).
fatales que producen daños importantes al niño, incluso la muerte.
La actitud de los padres (despreocupación por la gravedad de las Pregunta 97.- R: 4
lesiones, justificaciones poco lógicas para las lesiones detectadas, Existen numerosos síndromes genéticos y polimalformativos que se ma-
preocupación por la propia situación personal y no por la del niño, nifiestan con talla baja. Sin embargo, el síndrome de Klinefelter (47xxy)
etc.), la actitud del niño (ansiedad excesiva al hablarle y explorarle, se caracteriza por presentar talla alta junto con otras alteraciones (retra-
pasividad inadecuada para su edad, conducta agresiva, distancia so mental, alteraciones de la conducta, DM, alteraciones de la función
o proximidad excesivas con los padres) y la exploración física, son tiroidea, testes pequeños e infertilidad, etc.).
las herramientas que tenemos para sospechar el diagnóstico. De la
exploración, son indicadores muy sugerentes: ETS en niñas prepúbe- Recuerda las principales características de los otros síndromes:
res, embarazo en adolescente joven, lesiones genitales o anales sin • Síndrome de Turner (45x0): talla baja, Pterigium colli, ausencia de
traumatismo previo; son indicadores inespecíficos: dolor o sangrado desarrollo puberal, etc. Puede asociar cardiopatía y alteraciones
vaginal o rectal, enuresis, encopresis, dolor abdominal, lesiones renales.
traumáticas en distintas fases de evolución, lesiones en localizacio- • Síndrome de Down (95% casos, trisomía 21): hipotonía, hendiduras
nes que no corresponden con la actividad del niño ni su desarrollo palpebrales, aumento de piel en zona nucal, retraso mental variable.
psicomotor, fracturas raras en niños como la del acromión. Ante la Puede asociar cardiopatía (siendo típico el canal AV).
sospecha fundada, es obligatorio dar parte a la Autoridad Judicial • Síndrome de Silver-Russell: cuadro de talla baja que se manifiesta
y ponerse en contacto con los Servicios Sociales y de Protección y desde la etapa fetal y aspecto pseudohidrocefálico.
Atención al Menor, y ampliar el estudio con una serie ósea completa, • Síndrome de Noonan: talla baja, epicantus, hipertelorismo ocular, raíz
un examen del fondo de ojo, una TC craneal, entre otras. nasal hundida, hipertelorismo mamilar. Es característica la estenosis
pulmonar.
Pregunta 94.- R: 3
La CoA consiste en un estrechamiento de la luz aórtica. Recuerda que Pregunta 98.- R: 1
es más frecuente en niñas con síndrome de Turner. El tipo más común La causa más frecuente de convulsiones en el RN es la encefalopatía
es yuxtaductal (respuesta 1 correcta). hipoxicoisquémica. Las crisis suelen ser focales, no generalizadas,
Los niños suelen estar asintomáticos (respuesta 2 correcta) y la enferme- puesto que el SNC del neonato no está completamente mielinizado.
dad puede pasar desapercibida hasta la edad adulta. En la exploración El diagnóstico se basa en el EEG. El fenobarbital es el fármaco de pri-
es típico encontrar pulsos en extremidades inferiores disminuidos y mera elección. El pronóstico dependerá de la causa desencadenante
retrasados en comparación con los miembros superiores, lo que supo- (respuesta 1 falsa).
ne HTA en miembros superiores (respuesta 3 falsa) e incluso puede ser
mayor la TA en el miembro derecho que en el izquierdo, sobre todo si Pregunta 99.- R: 3
la coartación es proximal a la salida de la subclavia izquierda (respuesta La enuresis es la emisión involuntaria de orina en la ropa o en la cama
4 correcta). En la Rx de tórax son típicas las escotaduras costales en los después de la edad en que los niños deben tener control de esfínteres
bordes inferiores de las costillas o signo de Rösler (respuesta 5 correcta) (5 años de edad), en ausencia de patología orgánica. Su prevalencia es
y, a veces, el llamado signo del 3 o escotadura de la aorta en el lugar de mayor en niños que en niñas (respuesta 1 falsa). Existe un claro patrón
la coartación. En el ECG aparecen signos de hipertrofia de VI. familiar especialmente importante para la enuresis nocturna (respuesta
El tratamiento en niños es quirúrgico. 3 correcta).
El 90% de los casos son primarios o persistentes (el niño no ha llegado
Pregunta 95.- R: 2 nunca a tener un control completo de la orina) y se deben a un trastorno
Pregunta sobre el desarrollo puberal normal. Recuerda que la pubertad madurativo. Los casos secundarios son más frecuentes entre los 5 y los 8
en niñas se inicia a partir de los 9 años y en niños a partir de los 11. años de edad, y característicamente son transitorios y de buen pronóstico
El aumento del tamaño testicular en el varón es el primer signo de pu- (respuesta 2 falsa). La enuresis diurna es más frecuente en niñas, y casi
bertad (respuesta 2 correcta). En las niñas, el primer dato a la exploración siempre se debe al aplazamiento de la micción (hábito retentor) hasta
sugestivo de inicio de la pubertad, es la aparición de botón mamario. rebasar la capacidad de la vejiga (respuesta 4 falsa).
La ganancia de talla es más acusada en niños que en niñas. El tratamiento más correcto debe buscarse después de una evaluación
psicosocial y la exploración física. Lo más efectivo son las técnicas
Pregunta 96.- R: 3 conductuales. La desmopresina es efectiva pero, asimismo, tiene una
La telarquia prematura es el desarrollo mamario prematuro en niñas sin alta tasa de recaídas, y supone un riesgo de retención hídrica e hipo-
otros caracteres sexuales secundarios, sin aceleración de la velocidad natremia, por lo que se recomienda sólo en situaciones puntuales.

CTO Medicina • C/Francisco Silvela, 106 • 28002 - Madrid • Tfno. (0034) 91 782 43 30/33/34 • E-mail: secretaria@ctomedicina.com • www. ctomedicina. 17
Comentarios de Test a distancia 1.ª vuelta
Pediatría
Pregunta 100.- R: 2 sentación es entre los 6 meses y 5 años. Existen antecedentes familiares
El cuadro clínico hace referencia a un niño con una convulsión febril. La hasta en un 30% de los casos. El tratamiento en la fase aguda consiste
clínica típica consiste en fiebre alta, crisis tónico-clónica generalizada en la administración de diazepam intrarrectal y medidas para disminuir
de breve duración con un periodo postcrítico corto. La edad de pre- la hipertermia.

CTO Medicina • C/Francisco Silvela, 106 • 28002 - Madrid • Tfno. (0034) 91 782 43 30/33/34 • E-mail: secretaria@ctomedicina.com • www. ctomedicina.com 18

S-ar putea să vă placă și